SlideShare a Scribd company logo
1 of 843
Download to read offline
more than 1 million copies sold worldwide
®



                         12th EDITION
GMAT          REVIEW
                The only study guide with
                more than 800 past GMAT
                                                                               ®




                questions—and their
                answers—
                       —by the creators
                of the test.                                 ~The ~ L
                                                               ICIA
                                                          OFF uide
             THE OFFICIAL GUIDE FOR
                                                            G
             GMAT ® REVIEW, 12TH EDITION
             • Actual questions from past GMAT tests
             • Diagnostic section helps you assess where to focus
               your test-prep efforts
             • Insights into the GMAT exam that debunk test-taking myths



                                                                           ®
             From the Graduate Management Admission Council
more than 1 million copies sold worldwide
®



                         12th EDITION
GMAT          REVIEW
                The only study guide with
                more than 800 past GMAT
                                                                               ®




                questions—and their
                answers—
                       —by the creators
                of the test.                                 ~The ~ L
                                                               ICIA
                                                          OFF uide
             THE OFFICIAL GUIDE FOR
                                                            G
             GMAT ® REVIEW, 12TH EDITION
             • Actual questions from past GMAT tests
             • Diagnostic section helps you assess where to focus
               your test-prep efforts
             • Insights into the GMAT exam that debunk test-taking myths



                                                                           ®
             From the Graduate Management Admission Council
THE OFFICIAL GUIDE FOR GMAT® REVIEW, 12TH EDITION
Copyright © 2009 by the Graduate Management Admission Council®. All rights reserved.

Published by Wiley Publishing, Inc., Hoboken, New Jersey

No part of this publication may be reproduced, stored in a retrieval system or transmitted in any form
or by any means, electronic, mechanical, photocopying, recording, scanning or otherwise, except as
permitted under Sections 107 or 108 of the 1976 United States Copyright Act, without either the prior
written permission of the Publisher, or authorization through payment of the appropriate per-copy fee
to the Copyright Clearance Center, 222 Rosewood Drive, Danvers, MA 01923, (978) 750-8400, fax
(978) 646-8600, or on the web at www.copyright.com. Requests to the Publisher for permission should
be addressed to the Permissions Department, John Wiley & Sons, Inc., 111 River Street, Hoboken, NJ
07030, (201) 748-6011, fax (201) 748-6008, or online at http://www.wiley.com/go/permissions.

The publisher and the author make no representations or warranties with respect to the accuracy or
completeness of the contents of this work and specifically disclaim all warranties, including without
limitation warranties of fitness for a particular purpose. No warranty may be created or extended by
sales or promotional materials. The advice and strategies contained herein may not be suitable for every
situation. This work is sold with the understanding that the publisher is not engaged in rendering legal,
accounting, or other professional services. If professional assistance is required, the services of a
competent professional person should be sought. Neither the publisher nor the author shall be liable
for damages arising here from. The fact that an organization or Web site is referred to in this work as a
citation and/or a potential source of further information does not mean that the author or the publisher
endorses the information the organization or Web site may provide or recommendations it may make.
Further, readers should be aware that Internet Web sites listed in this work may have changed or
disappeared between when this work was written and when it is read.

Trademarks: Wiley, the Wiley Publishing logo, and related trademarks are trademarks or registered
trademarks of John Wiley & Sons, Inc. and/or its affiliates. GMAC®, GMAT®, GMAT CAT®,
Graduate Management Admission Council®, and Graduate Management Admission Test® are
registered trademarks of the Graduate Management Admission Council® (GMAC®). Creating Access
to Graduate Business Educationsm is a service mark of the Graduate Management Admission Council®.
All other trademarks are the property of their respective owners. Wiley Publishing, Inc. is not
associated with any product or vendor mentioned in this book.

For general information on our other products and services or to obtain technical support please
contact our Customer Care Department within the U.S. at (877) 762-2974, outside the U.S. at (317)
572-3993 or fax (317) 572-4002.

Wiley also publishes its books in a variety of electronic formats. Some content that appears in print
may not be available in electronic books. For more information about Wiley products, please visit our
Web site at www.wiley.com.

Library of Congress Control Number: 2008940699

ISBN: 978-0-470-44974-5

Printed in the United States of America

10 9 8 7 6 5 4 3 2 1

Book production by Wiley Publishing, Inc. Composition Services
Charles Forster, Designer
Mike Wilson, Production Designer
Table of Contents   1.0    What Is the GMAT®?                           6
                    1.1    Why Take the GMAT® Test?                     7
                    1.2    GMAT® Test Format                            8
                    1.3    What Is the Content of the Test Like?       10
                    1.4    Quantitative Section                        10
                    1.5    Verbal Section                              10
                    1.6    What Computer Skills Will I Need?           11
                    1.7    What Are the Test Centers Like?             11
                    1.8    How Are Scores Calculated?                  11
                    1.9    Analytical Writing Assessment Scores        12
                    1.10   Test Development Process                    13

                    2.0    How to Prepare                              14
                    2.1    How Can I Best Prepare to Take the Test?    15
                    2.2    What About Practice Tests?                  15
                    2.3    How Should I Use the Diagnostic Test?       16
                    2.4    Where Can I Get Additional Practice?        16
                    2.5    General Test-Taking Suggestions             16

                    3.0    Diagnostic Test                             18
                    3.1    Quantitative Sample Questions               20
                    3.2    Verbal Sample Questions                     27
                    3.3    Quantitative and Verbal Answer Keys         45
                    3.4    Interpretive Guide                          45
                    3.5    Quantitative Answer Explanations            46
                    3.6    Verbal Answer Explanations                  65

                    4.0    Math Review                                106
                    4.1    Arithmetic                                 108
                    4.2    Algebra                                    120
                    4.3    Geometry                                   127
                    4.4    Word Problems                              140

                    5.0    Problem Solving                            148
                    5.1    Test-Taking Strategies                     150
                    5.2    The Directions                             150
                    5.3    Sample Questions                           152
                    5.4    Answer Key                                 186
                    5.5    Answer Explanations                        188

                    6.0    Data Sufficiency                            266
                    6.1    Test-Taking Strategies                     268
                    6.2    The Directions                             270
                    6.3    Sample Questions                           272
                    6.4    Answer Key                                 289
                    6.5    Answer Explanations                        290

                    7.0    Reading Comprehension                      352
                    7.1    What Is Measured                           354
                    7.2    Test-Taking Strategies                     356
                    7.3    The Directions                             357
                    7.4    Sample Questions                           358
                    7.5    Answer Key                                 408
                    7.6    Answer Explanations                        409
8.0    Critical Reasoning                             482
      8.1    What Is Measured                               484
      8.2    Test-Taking Strategies                         484
      8.3    The Directions                                 485
      8.4    Sample Questions                               486
      8.5    Answer Key                                     525
      8.6    Answer Explanations                            526

      9.0    Sentence Correction                            650
      9.1    Basic English Grammar Rules                    651
      9.2    Study Suggestions                              656
      9.3    What Is Measured                               656
      9.4    Test-Taking Strategies                         656
      9.5    The Directions                                 657
      9.6    Sample Questions                               658
      9.7    Answer Key                                     684
      9.8    Answer Explanations                            685

      10.0   Analytical Writing Assessment                  758
      10.1   What Is Measured                               759
      10.2   Test-Taking Strategies                         760
      10.3   The Directions                                 761
      10.4   GMAT® Scoring Guide: Analysis of an Issue      762
      10.5   Sample: Analysis of an Issue                   764
      10.6   Analysis of an Issue Sample Topics             769
      10.7   GMAT® Scoring Guide: Analysis of an Argument   790
      10.8   Sample: Analysis of an Argument                792
      10.9   Analysis of an Argument Sample Topics          796

Appendix A   Percentile Ranking Tables                      828
Appendix B   Answer Sheets                                  834
             Diagnostic Answer Sheet                        835
             Problem Solving Answer Sheet                   836
             Data Sufficiency Answer Sheet                   837
             Reading Comprehension Answer Sheet             838
             Critical Reasoning Answer Sheet                839
             Sentence Correction Answer Sheet               840
Dear Future Business Leader,

By using this book to prepare for the GMAT® test, you are taking a very important step toward
gaining admission to a high-quality business or management program and achieving a rewarding
career in management. I applaud your decision.

The Graduate Management Admission Council® developed the GMAT test more than 50 years
ago to help leading graduate schools of business and management choose the applicants who best
suit their programs. Today, the test is used by more than 1,800 graduate programs and is given to
test takers daily in more than 110 countries around the world. Programs that use GMAT scores in
selective admissions have helped establish the MBA degree as a hallmark of excellence worldwide.

Why do GMAT scores matter so much? Other admissions factors—such as work experience,
grades, admissions essays, and interviews—can say something about who you are and what you have
done in your career, but only your GMAT scores can tell schools how you are likely to perform
academically in the business school courses that are fundamental to the MBA degree. In fact, the
test has been proven reliable as a predictor of academic performance for more than half a century.

In other words, business schools that require you to take the GMAT really care about the
quality of their student body. And excellent MBA students mean a stronger MBA program, a
more enriching learning environment, and a more valuable degree for you to take into the business
world. By enrolling in a school that uses the GMAT test for your graduate business degree, you
will maximize the value of your degree, and that value will pay off in many ways, throughout
your career.

I wish you great success in preparing for this important next step in your professional education,
and I wish you a very rewarding management career.

Sincerely




David A. Wilson
President and CEO
Graduate Management Admission Council®
1.0   What Is the GMAT®?




6
1.0 What Is the GMAT®?




1.0 What Is the GMAT®?
   The Graduate Management Admission Test® (GMAT®) is a standardized, three-part test delivered
   in English. The test was designed to help admissions officers evaluate how suitable individual
   applicants are for their graduate business and management programs. It measures basic verbal,
   mathematical, and analytical writing skills that a test taker has developed over a long period of time
   through education and work.

   The GMAT test does not a measure a person’s knowledge of specific fields of study. Graduate
   business and management programs enroll people from many different undergraduate and work
   backgrounds, so rather than test your mastery of any particular subject area, the GMAT test will
   assess your acquired skills. Your GMAT score will give admissions officers a statistically reliable
   measure of how well you are likely to perform academically in the core curriculum of a graduate
   business program.

   Of course, there are many other qualifications that can help people succeed in business school and in
   their careers—for instance, job experience, leadership ability, motivation, and interpersonal skills.
   The GMAT test does not gauge these qualities. That is why your GMAT score is intended to be
   used as one standard admissions criterion among other, more subjective, criteria, such as admissions
   essays and interviews.

1.1 Why Take the GMAT® Test?
   GMAT scores are used by admissions officers in roughly 1,800 graduate business and management
   programs worldwide. Schools that require prospective students to submit GMAT scores in the
   application process are generally interested in
   admitting the best-qualified applicants for their
   programs, which means that you may find a more                               -vs-     FACT
   beneficial learning environment at schools that
   require GMAT scores as part of your application.             – If I don’t score in the 90th
                                                                     percentile, I won’t get into
   Because the GMAT test gauges skills that are                      any school I choose.
   important to successful study of business and
                                                                F – Very few people get very
   management at the graduate level, your scores will
                                                                    high scores.
   give you a good indication of how well prepared
   you are to succeed academically in a graduate                Fewer than 50 of the more than 200,000
   management program; how well you do on the test              people taking the GMAT test each year
                                                                get a perfect score of 800. Thus, while
   may also help you choose the business schools to             you may be exceptionally capable, the odds
   which you apply. Furthermore, the percentile table           are against your achieving a perfect score.
   you receive with your scores will tell you how your          Also, the GMAT test is just one piece of
                                                                your application packet. Admissions officers
   performance on the test compares to the                      use GMAT scores in conjunction with
   performance of other test takers, giving you one             undergraduate records, application essays,
   way to gauge your competition for admission to               interviews, letters of recommendation, and
                                                                other information when deciding whom to
   business school.                                             accept into their programs.




                                                                                                               7
The Official Guide for GMAT® Review 12th Edition




             Schools consider many different aspects of an application before making an admissions decision, so
             even if you score well on the GMAT test, you should contact the schools that interest you to learn
             more about them and to ask about how they use GMAT scores and other admissions criteria (such
             as your undergraduate grades, essays, and letters of recommendation) to evaluate candidates for
             admission. School admissions offices, school Web sites, and materials published by the school are
             the best sources for you to tap when you are doing research about where you might want to go to
             business school.

             For more information about how schools should use GMAT scores in admissions decisions,
             please read Appendix A of this book. For more information on the GMAT, registering to take
             the test, sending your scores to schools, and applying to business school, please visit our Web site
             at www.mba.com.

     1.2 GMAT® Test Format
             The GMAT test consists of four separately timed sections (see the table on the next page).
             You start the test with two 30-minute Analytical Writing Assessment (AWA) questions that
             require you to type your responses using the computer keyboard. The writing section is followed by
             two 75-minute, multiple-choice sections: the Quantitative and Verbal sections of the test.

             The GMAT is a computer-adaptive test (CAT),
             which means that in the multiple-choice sections                                 -vs-   FACT
             of the test, the computer constantly gauges how
             well you are doing on the test and presents you                  – Getting an easier question
             with questions that are appropriate to your ability                means I answered the last one
             level. These questions are drawn from a huge pool                  wrong.
             of possible test questions. So, although we talk
             about the GMAT as one test, the GMAT test you                 F – Getting an easier question
             take may be completely different from the test of                  does not necessarily mean you
                                                                               got the previous question
             the person sitting next to you.
                                                                               wrong.
             Here’s how it works. At the start of each GMAT                To ensure that everyone receives the same
             multiple-choice section (Verbal and Quantitative),            content, the test selects a specific number
                                                                           of questions of each type. The test may call
             you will be presented with a question of moderate             for your next question to be a relatively
             difficulty. The computer uses your response to that            hard problem-solving item involving
             first question to determine which question to                  arithmetic operations. But, if there are no
                                                                           more relatively difficult problem-solving
             present next. If you respond correctly, the test              items involving arithmetic, you might be
             usually will give you questions of increasing                 given an easier item.
             difficulty. If you respond incorrectly, the next               Most people are not skilled at estimating
             question you see usually will be easier than the one          item difficulty, so don’t worry when taking
             you answered incorrectly. As you continue to                  the test or waste valuable time trying to
                                                                           determine the difficulty of the questions
             respond to the questions presented, the computer              you are answering.
             will narrow your score to the number that best
             characterizes your ability. When you complete
             each section, the computer will have an accurate
             assessment of your ability.




8
1.2 What Is the GMAT®? GMAT® Test Format




Because each question is presented on the basis of your answers to all previous questions, you must
answer each question as it appears. You may not skip, return to, or change your responses to
previous questions. Random guessing can significantly lower your scores. If you do not know the
answer to a question, you should try to eliminate as many choices as possible, then select the answer
you think is best. If you answer a question incorrectly by mistake—or correctly by lucky guess—
your answers to subsequent questions will lead you back to questions that are at the appropriate level
of difficulty for you.

Each multiple-choice question used in the GMAT test has been thoroughly reviewed by
professional test developers. New multiple-choice questions are tested each time the test is
administered. Answers to trial questions are not counted in the scoring of your test, but the trial
questions are not identified and could appear anywhere in the test. Therefore, you should try to do
your best on every question.

The test includes the types of questions found in this guide, but the format and presentation of the
questions are different on the computer. When you take the test:

  • Only one question at a time is presented on the computer screen.
  • The answer choices for the multiple-choice questions will be preceded by circles, rather than by
    letters.
  • Different question types appear in random order in the multiple-choice sections of the test.
  • You must select your answer using the computer.
  • You must choose an answer and confirm your choice before moving on to the next question.
  • You may not go back to change answers to previous questions.



                               Format of the GMAT®
                                                Questions          Timing
              Analytical Writing
                Analysis of an Argument             1                 30 min.
                Analysis of an Issue                1                 30 min.
              Optional break                                           5 min.
              Quantitative                         37                 75 min.
                Problem Solving
                Data Sufficiency
              Optional break                                           5 min.
              Verbal                               41                 75 min.
                Reading Comprehension
                Critical Reasoning
                Sentence Correction

                                              Total Time:       210–220 min.




                                                                                                         9
The Official Guide for GMAT® Review 12th Edition




     1.3 What Is the Content of the Test Like?
             It is important to recognize that the GMAT test evaluates skills and abilities developed over a
             relatively long period of time. Although the sections contain questions that are basically verbal and
             mathematical, the complete test provides one method of measuring overall ability.

             Keep in mind that although the questions in this guide are arranged by question type and ordered
             from easy to difficult, the test is organized differently. When you take the test, you may see different
             types of questions in any order.

     1.4 Quantitative Section
             The GMAT Quantitative section measures your ability to reason quantitatively, solve quantitative
             problems, and interpret graphic data.

             Two types of multiple-choice questions are used in the Quantitative section:

                • Problem solving
                • Data sufficiency

             Problem solving and data sufficiency questions are intermingled throughout the Quantitative
             section. Both types of questions require basic knowledge of:

                • Arithmetic
                • Elementary algebra
                • Commonly known concepts of geometry

             To review the basic mathematical concepts that will be tested in the GMAT Quantitative
             questions, see the math review in chapter 4. For test-taking tips specific to the question types in
             the Quantitative section of the GMAT test, sample questions, and answer explanations, see
             chapters 5 and 6.

     1.5 Verbal Section
             The GMAT Verbal section measures your ability to read and comprehend written material, to
             reason and evaluate arguments, and to correct written material to conform to standard written
             English. Because the Verbal section includes reading sections from several different content areas,
             you may be generally familiar with some of the material; however, neither the reading passages nor
             the questions assume detailed knowledge of the topics discussed.

             Three types of multiple-choice questions are used in the Verbal section:

                • Reading comprehension
                • Critical reasoning
                • Sentence correction

             These question types are intermingled throughout the Verbal section.
10
1.6 What Is the GMAT®? What Computer Skills Will I Need?




   For test-taking tips specific to each question type in the Verbal section, sample questions, and
   answer explanations, see chapters 7 through 9.

1.6 What Computer Skills Will I Need?
   You only need minimal computer skills to take the GMAT Computer-Adaptive Test (CAT). You
   will be required to type your essays on the computer keyboard using standard word-processing
   keystrokes. In the multiple-choice sections, you will select your responses using either your mouse or
   the keyboard.

   To learn more about the specific skills required to take the GMAT CAT, download the free test-
   preparation software available at www.mba.com.

1.7 What Are the Test Centers Like?
   The GMAT test is administered at a test center providing the quiet and privacy of individual
   computer workstations. You will have the opportunity to take two five-minute breaks—one after
   completing the essays and another between the Quantitative and Verbal sections. An erasable
   notepad will be provided for your use during the test.

1.8 How Are Scores Calculated?
   Your GMAT scores are determined by:

     • The number of questions you answer
     • Whether you answer correctly or incorrectly
     • The level of difficulty and other statistical characteristics of each question

   Your Verbal, Quantitative, and Total GMAT scores are determined by a complex mathematical
   procedure that takes into account the difficulty of the questions that were presented to you and how
   you answered them. When you answer the easier questions correctly, you get a chance to answer
   harder questions—making it possible to earn a higher score. After you have completed all the
   questions on the test—or when your time is up—the computer will calculate your scores. Your
   scores on the Verbal and Quantitative sections are combined to produce your Total score. If you have
   not responded to all the questions in a section (37 Quantitative questions or 41 Verbal questions),
   your score is adjusted, using the proportion of questions answered.

   Appendix A contains the 2007 percentile ranking tables that explain how your GMAT scores
   compare with scores of other 2007 GMAT test takers.




                                                                                                              11
The Official Guide for GMAT® Review 12th Edition




     1.9 Analytical Writing Assessment Scores
             The Analytical Writing Assessment consists of two writing tasks: Analysis of an Issue and Analysis
             of an Argument. The responses to each of these tasks are scored on a 6-point scale, with 6 being the
             highest score and 1, the lowest. A score of zero (0) is given to responses that are off-topic, are in a
             foreign language, merely attempt to copy the topic, consist only of keystroke characters, or are
             blank.

             The readers who evaluate the responses are college and university faculty members from various
             subject matter areas, including management education. These readers read holistically—that is, they
             respond to the overall quality of your critical thinking and writing. (For details on how readers are
             qualified, visit www.mba.com.) In addition, responses may be scored by an automated scoring
             program designed to reflect the judgment of expert readers.

             Each response is given two independent ratings. If the ratings differ by more than a point, a third
             reader adjudicates. (Because of ongoing training and monitoring, discrepant ratings are rare.)

             Your final score is the average (rounded to the nearest half point) of the four scores independently
             assigned to your responses—two scores for the Analysis of an Issue and two for the Analysis of an
             Argument. For example, if you earned scores of 6 and 5 on the Analysis of an Issue and 4 and 4 on
             the Analysis of an Argument, your final score would be 5: (6 + 5 + 4 + 4) ÷ 4 = 4.75, which rounds
             up to 5.

             Your Analytical Writing Assessment scores are computed and reported separately from the
             multiple-choice sections of the test and have no effect on your Verbal, Quantitative, or Total scores.
             The schools that you have designated to receive your scores may receive your responses to the
             Analytical Writing Assessment with your score report. Your own copy of your score report will not
             include copies of your responses.




12
1.10 What Is the GMAT®? Test Development Process




1.10 Test Development Process
    The GMAT test is developed by experts who use standardized procedures to ensure high-quality,
    widely appropriate test material. All questions are subjected to independent reviews and are revised
    or discarded as necessary. Multiple-choice questions are tested during GMAT test administrations.
    Analytical Writing Assessment tasks are tried out on first-year business school students and then
    assessed for their fairness and reliability. For more information on test development, see www.
    mba.com.




                                                                                                             13
2.0 How to Prepare




14
2.0 How to Prepare




2.0 How to Prepare
2.1 How Can I Best Prepare to Take the Test?
   We at the Graduate Management Admission Council® (GMAC®) firmly believe that the test-
   taking skills you can develop by using this guide—and the Verbal and Quantitative guides, if you
   want additional practice—are all you need to perform your best when you take the GMAT® test.
   By answering questions that have appeared on the GMAT test before, you will gain experience with
   the types of questions you may see on the test when you take it. As you practice with this guide, you
   will develop confidence in your ability to reason through the test questions. No additional
   techniques or strategies are needed to do well on the standardized test if you develop a practical
   familiarity with the abilities it requires. Simply by practicing and understanding the concepts that
   are assessed on the test, you will learn what you need to know to answer the questions correctly.

2.2 What About Practice Tests?
   Because a computer-adaptive test cannot be presented in paper form, we have created GMATPrep
   software to help you prepare for the test. The software is available for download at no charge for
   those who have created a user profile on www.mba.com. It is also provided on a disk, by request, to
   anyone who has registered for the GMAT test. The software includes two practice GMAT tests
   plus additional practice questions, information about the test, and tutorials to help you become
   familiar with how the GMAT test will appear on the computer screen at the test center.

   We recommend that you download the software as
   you start to prepare for the test. Take one practice                           -vs-   FACT
   test to familiarize yourself with the test and to get
   an idea of how you might score. After you have                 – You need very advanced
   studied using this book, and as your test date                   math skills to get a high
   approaches, take the second practice test to                     GMAT score.
   determine whether you need to shift your focus
   to other areas you need to strengthen.                      F – The math skills tested on the
                                                                   GMAT test are quite basic.
   If you complete all the questions in this guide and         The GMAT test only requires basic
   think you would like additional practice, you may           quantitative analytic skills. You should
   purchase The Official Guide for GMAT® Verbal                 review the math skills (algebra, geometry,
                                                               basic arithmetic) presented in this book, but
   Review or The Official Guide for GMAT®                       the required skill level is low. The difficulty
   Quantitative Review at www.mba.com.                         of GMAT Quantitative questions stems
                                                               from the logic and analysis used to solve
                                                               the problems and not the underlying math
   Note: There may be some overlap between this                skills.
   book and the review sections of the GMATPrep
   software.




                                                                                                                15
The Official Guide for GMAT® Review 12th Edition




     2.3 How Should I Use the Diagnostic Test?
             This book contains a Diagnostic Test to help you determine the types of questions that you need to
             practice most. You should take the Diagnostic Test around the same time that you take the first
             electronic sample test (using the test-preparation software). The Diagnostic Test will give you a
             rating—below average, average, above average, or excellent—of your skills in each type of GMAT
             test question. These ratings will help you identify areas to focus on as you prepare for the
             GMAT test.

             Use the results of the Diagnostic Test to help you select the right chapter of this book to start with.
             Next, read the introductory material carefully, and answer the sample questions in that chapter.
             Make sure you follow the directions for each type of question and try to work as quickly and as
             efficiently as possible. Then review the explanations for the correct answers, spending as much time
             as necessary to familiarize yourself with the range of questions or problems presented.

     2.4 Where Can I Get Additional Practice?
             If you find you would like additional practice with the Verbal section of the test, The Official Guide
             for GMAT® Verbal Review is available for purchase at www.mba.com. If you want more practice
             with the Quantitative section, The Official Guide for GMAT® Quantitative Review is also available
             for purchase at www.mba.com.

     2.5 General Test-Taking Suggestions
             Specific test-taking strategies for individual question types are presented later in this book. The
             following are general suggestions to help you perform your best on the test.

     1. Use your time wisely.
             Although the GMAT test stresses accuracy more than speed, it is important to use your time wisely.
             On average, you will have about 1¾ minutes for each verbal question and about two minutes for
             each quantitative question. Once you start the test, an onscreen clock will continuously count the
             time you have left. You can hide this display if you want, but it is a good idea to check the clock
             periodically to monitor your progress. The clock will automatically alert you when five minutes
             remain in the allotted time for the section you are working on.

     2. Answer practice questions ahead of time.
             After you become generally familiar with all question types, use the sample questions in this book
             to prepare for the actual test. It may be useful to time yourself as you answer the practice questions
             to get an idea of how long you will have for each question during the actual GMAT test as well as
             to determine whether you are answering quickly enough to complete the test in the time allotted.

     3. Read all test directions carefully.
             The directions explain exactly what is required to answer each question type. If you read hastily, you
             may miss important instructions and lower your scores. To review directions during the test, click


16
2.5 How to Prepare General Test-Taking Suggestions




      on the Help icon. But be aware that the time you spend reviewing directions will count against the
      time allotted for that section of the test.

4. Read each question carefully and
   thoroughly.                                                                      -vs-   FACT
      Before you answer a multiple-choice question,                 – It is more important to
      determine exactly what is being asked, then                     respond correctly to the test
      eliminate the wrong answers and select the best                 questions than it is to finish
      choice. Never skim a question or the possible                   the test.
      answers; skimming may cause you to miss
      important information or nuances.                          F – There is a severe penalty
                                                                     for not completing the
                                                                     GMAT test.
5. Do not spend too much time on any one
   question.                                                     If you are stumped by a question, give it
                                                                 your best guess and move on. If you guess
      If you do not know the correct answer, or if the           incorrectly, the computer program will likely
                                                                 give you an easier question, which you are
      question is too time-consuming, try to eliminate           likely to answer correctly, and the computer
      choices you know are wrong, select the best of the         will rapidly return to giving you questions
                                                                 matched to your ability. If you don’t finish
      remaining answer choices, and move on to the next          the test, your score will be reduced greatly.
      question. Try not to worry about the impact on             Failing to answer five verbal questions, for
      your score—guessing may lower your score, but              example, could reduce your score from
                                                                 the 91st percentile to the 77th percentile.
      not finishing the section will lower your score             Pacing is important.
      more.

      Bear in mind that if you do not finish a section in
      the allotted time, you will still receive a score.
                                                                                    -vs-   FACT
6. Confirm your answers ONLY when you are                            – The first 10 questions are
   ready to move on.                                                  critical and you should invest
                                                                      the most time on those.
      Once you have selected your answer to a multiple-
      choice question, you will be asked to confirm it.           F – All questions count.
      Once you confirm your response, you cannot go
      back and change it. You may not skip questions,            It is true that the computer-adaptive testing
                                                                 algorithm uses the first 10 questions to
      because the computer selects each question on the          obtain an initial estimate of your ability;
      basis of your responses to preceding questions.            however, that is only an initial estimate. As
                                                                 you continue to answer questions, the
                                                                 algorithm self-corrects by computing an
                                                                 updated estimate on the basis of all the
7. Plan your essay answers before you begin                      questions you have answered, and then
   to write.                                                     administers items that are closely matched
                                                                 to this new estimate of your ability. Your
      The best way to approach the two writing tasks             final score is based on all your responses
      that comprise the Analytical Writing Assessment            and considers the difficulty of all the
                                                                 questions you answered. Taking additional
      is to read the directions carefully, take a few            time on the first 10 questions will not game
      minutes to think about the question, and plan a            the system and can hurt your ability to
                                                                 finish the test.
      response before you begin writing. Take care to
      organize your ideas and develop them fully, but
      leave time to reread your response and make any
      revisions that you think would improve it.

                                                                                                                 17
3.0 Diagnostic Test




18
3.0 Diagnostic Test




3.0 Diagnostic Test
    Like the practice sections later in the book, the Diagnostic Test uses questions from real GMAT®
    tests. The purpose of the Diagnostic Test is to help you determine how skilled you are in answering
    each of the five types of questions on the GMAT test: data sufficiency, problem solving, reading
    comprehension, critical reasoning, and sentence correction.

    Scores on the Diagnostic Test are designed to help you answer the question, “If all the questions on
    the GMAT test were like the questions in this section, how well would I do?” Your scores are
    classified as being excellent, above average, average, or below average, relative to the scores of other test
    takers. You can use this information to focus your test-preparation activities.

  Instructions
      1. Take your time answering these questions. The Diagnostic Test is not timed.
      2. If you are stumped by a question, you should guess and move on, just like you should
         do on the real GMAT test.
      3. You can take one segment at a time, if you want. It is better to finish an entire section
         (Quantitative or Verbal) in one sitting, but this is not a requirement.
      4. You can go back and change your answers in the Diagnostic Test.
      5. After you take the test, check your answers using the answer key that follows the test.
         The number of correct answers is your raw score.
      6. Convert your raw score, using the table provided.
    Note: The Diagnostic Test is designed to give you guidance on how to prepare for the GMAT test;
    however, a strong score on one type of question does not guarantee that you will perform as well on
    the real GMAT test. The statistical reliability of scores on the Diagnostic Test ranges from 0.75 to
    0.89, and the subscale classification is about 85%–90% accurate, meaning that your scores on the
    Diagnostic Test are a good, but not perfect, measure of how you are likely to perform on the real
    test. Use the tests on the free online software to obtain a good estimate of your expected GMAT
    Verbal, Quantitative, and Total scores.

    You should not compare the number of questions you got right in each section. Instead, you should
    compare how your responses are rated in each section.




                                                                                                                   19
The Official Guide for GMAT® Review 12th Edition




 3.1 Quantitative Sample Questions
     Problem Solving
     Solve the problem and indicate the best of the answer choices given.
     Numbers: All numbers used are real numbers.
     Figures:   All figures accompanying problem solving questions are intended to provide information
                useful in solving the problems. Figures are drawn as accurately as possible. Exceptions will be
                clearly noted. Lines shown as straight are straight, and lines that appear jagged are also
                straight. The positions of points, angles, regions, etc., exist in the order shown, and angle
                measures are greater than zero. All figures lie in a plane unless otherwise indicated.



1.   Last month a certain music club offered a discount to    4.   Among a group of 2,500 people, 35 percent invest in
     preferred customers. After the first compact disc              municipal bonds, 18 percent invest in oil stocks, and
     purchased, preferred customers paid $3.99 for each            7 percent invest in both municipal bonds and oil
     additional compact disc purchased. If a preferred             stocks. If 1 person is to be randomly selected from
     customer purchased a total of 6 compact discs and             the 2,500 people, what is the probability that the
     paid $15.95 for the first compact disc, then the dollar        person selected will be one who invests in municipal
     amount that the customer paid for the 6 compact               bonds but NOT in oil stocks?
     discs is equivalent to which of the following?
                                                                   (A)
     (A)    5(4.00) + 15.90
     (B)    5(4.00) + 15.95                                              7
                                                                   (B)
     (C)    5(4.00) + 16.00                                              25
     (D)    5(4.00 – 0.01) + 15.90                                       7
                                                                   (C)
     (E)    5(4.00 – 0.05) + 15.95                                       20
                                                                         21
                                                                   (D)
2.   The average (arithmetic mean) of the integers from                  50
     200 to 400, inclusive, is how much greater than the                 27
                                                                   (E)
     average of the integers from 50 to 100, inclusive?                  50
     (A)    150
     (B)    175                                               5.   A closed cylindrical tank contains 36π cubic feet of
     (C)    200                                                    water and is filled to half its capacity. When the tank
                                                                   is placed upright on its circular base on level ground,
     (D)    225
                                                                   the height of the water in the tank is 4 feet. When the
     (E)    300
                                                                   tank is placed on its side on level ground, what is the
                                                                   height, in feet, of the surface of the water above the
3.   The sequence a1, a2, a3,...,an,... is such that               ground?

     an =             for all n ≥ 3. If a3 = 4 and                 (A)   2
                                                                   (B)   3
     a5 = 20, what is the value of a6 ?
                                                                   (C)   4
     (A)    12                                                     (D)   6
     (B)    16                                                     (E)   9
     (C)    20
     (D)    24
     (E)    28


20
3.1 Diagnostic Test Quantitative Sample Questions




6.   A marketing firm determined that, of 200 households                    1
                                                                     (A)
     surveyed, 80 used neither Brand A nor Brand B soap,                   4
     60 used only Brand A soap, and for every household                    1
     that used both brands of soap, 3 used only Brand B              (B)
                                                                           2
     soap. How many of the 200 households surveyed used
                                                                           2
     both brands of soap?                                            (C)
                                                                           3
     (A)   15
                                                                     (D)   2
     (B)   20
                                                                     (E)   4
     (C)   30
     (D)   40
                                                               9.    A researcher computed the mean, the median, and the
     (E)   45
                                                                     standard deviation for a set of performance scores.
                                                                     If 5 were to be added to each score, which of these
7.   A certain club has 10 members, including Harry. One             three statistics would change?
     of the 10 members is to be chosen at random to be
     the president, one of the remaining 9 members is to             (A)   The mean only
     be chosen at random to be the secretary, and one of             (B)   The median only
     the remaining 8 members is to be chosen at random               (C)   The standard deviation only
     to be the treasurer. What is the probability that Harry         (D)   The mean and the median
     will be either the member chosen to be the secretary            (E)   The mean and the standard deviation
     or the member chosen to be the treasurer?
            1
     (A)                                                                                 y°
           720
           1
     (B)                                                                       x°                        z°
           80
           1
     (C)
           10
            1
     (D)                                                                            v°
            9                                                                                      w°
            1
     (E)
            5

                                                               10.   In the figure shown, what is the value of
8.   If a certain toy store’s revenue in November was                v+x+y+z+w?
      2
         of its revenue in December and its revenue in               (A)    45
      5
                    1                                                (B)    90
     January was of its revenue in November, then the
                    4                                                (C)   180
     store’s revenue in December was how many times the
                                                                     (D)   270
     average (arithmetic mean) of its revenues in November
     and January?                                                    (E)   360




                                                                                                                         21
The Official Guide for GMAT® Review 12th Edition




11.   Of the three-digit integers greater than 700, how many    15.   The product of all the prime numbers less than 20 is
      have two digits that are equal to each other and the            closest to which of the following powers of 10 ?
      remaining digit different from the other two?
                                                                      (A)      109
      (A)   90                                                        (B)      108
      (B)   82                                                        (C)      107
      (C)   80                                                        (D)      106
      (D)   45                                                        (E)      105
      (E)   36
                                                                16.   If    3 − 2x = 2x + 1 , then 4x2 =
12.   Positive integer y is 50 percent of 50 percent of
                                                                      (A)      1
      positive integer x, and y percent of x equals 100. What
      is the value of x ?                                             (B)      4
                                                                      (C)      2 − 2x
      (A)        50                                                   (D)      4x − 2
      (B)     100                                                     (E)      6x − 1
      (C)    200
      (D)   1,000                                                                  16
                                                                17.   If n =          , what is the value of   ?
      (E)   2,000                                                                  81
                                                                               1
                                                s                     (A)
13.   If s and t are positive integers such that = 64.12,                      9
                                                t
      which of the following could be the remainder when                       1
                                                                      (B)
      s is divided by t ?                                                      4
                                                                               4
      (A)    2                                                        (C)
                                                                               9
      (B)    4                                                                 2
                                                                      (D)
      (C)    8                                                                 3
      (D)   20                                                                 9
                                                                      (E)
      (E)   45                                                                 2

14.   Of the 84 parents who attended a meeting at a             18.   If n is the product of the integers from 1 to 8,
      school, 35 volunteered to supervise children during             inclusive, how many different prime factors greater
      the school picnic and 11 volunteered both to supervise          than 1 does n have?
      children during the picnic and to bring refreshments to
      the picnic. If the number of parents who volunteered            (A)      Four
      to bring refreshments was 1.5 times the number of               (B)      Five
      parents who neither volunteered to supervise children           (C)      Six
      during the picnic nor volunteered to bring                      (D)      Seven
      refreshments, how many of the parents volunteered to            (E)      Eight
      bring refreshments?

      (A)   25                                                  19.   If k is an integer and 2 < k < 7, for how many different
      (B)   36                                                        values of k is there a triangle with sides of lengths 2,
                                                                      7, and k ?
      (C)   38
      (D)   42                                                        (A)      One
      (E)   45                                                        (B)      Two
                                                                      (C)      Three
                                                                      (D)      Four
                                                                      (E)      Five


22
3.1 Diagnostic Test Quantitative Sample Questions




20.   A right circular cone is inscribed in a hemisphere so     23.   If the positive integer x is a multiple of 4 and the
      that the base of the cone coincides with the base of            positive integer y is a multiple of 6, then xy must be a
      the hemisphere. What is the ratio of the height of the          multiple of which of the following?
      cone to the radius of the hemisphere?
                                                                       I.   8
      (A)                                                             II. 12
                                                                      III. 18
      (B)   1:1
            1                                                         (A)   II only
      (C)     :1
            2                                                         (B)   I and II only
      (D)                                                             (C)   I and III only
      (E)   2:1                                                       (D)   II and III only
                                                                      (E)   I, II, and III
21.   John deposited $10,000 to open a new savings
      account that earned 4 percent annual interest,            24.   Aaron will jog from home at x miles per hour and then
      compounded quarterly. If there were no other                    walk back home by the same route at y miles per hour.
      transactions in the account, what was the amount of             How many miles from home can Aaron jog so that he
      money in John’s account 6 months after the account              spends a total of t hours jogging and walking?
      was opened?
                                                                            xt
                                                                      (A)
      (A)   $10,100                                                         y
      (B)   $10,101                                                         x+t
      (C)   $10,200                                                   (B)
                                                                             xy
      (D)   $10,201
                                                                             xy t
      (E)   $10,400                                                   (C)
                                                                            x+y

22.   A container in the shape of a right circular cylinder                 x+y+t
                                                                      (D)
         1                                                                    xy
      is full of water. If the volume of water in the
         2
      container is 36 cubic inches and the height of the                    y+t t
                                                                      (E)      –
      container is 9 inches, what is the diameter of the base                x   y
      of the cylinder, in inches?

            16
      (A)
            9π

             4
      (B)
              π

            12
      (C)
             π

                2
      (D)
                π

                    2
      (E)   4
                    π




                                                                                                                             23
The Official Guide for GMAT® Review 12th Edition




     Data Sufficiency
     Each data sufficiency problem consists of a question and two statements, labeled (1) and (2), which
     contain certain data. Using these data and your knowledge of mathematics and everyday facts (such as
     the number of days in July or the meaning of the word counterclockwise), decide whether the data
     given are sufficient for answering the question and then indicate one of the following answer choices:
     A     Statement (1) ALONE is sufficient, but statement (2) alone is not sufficient.
     B     Statement (2) ALONE is sufficient, but statement (1) alone is not sufficient.
     C     BOTH statements TOGETHER are sufficient, but NEITHER statement ALONE is sufficient.
     D     EACH statement ALONE is sufficient.
     E     Statements (1) and (2) TOGETHER are not sufficient.
     Note: In data sufficiency problems that ask for the value of a quantity, the data given in the statements
     are sufficient only when it is possible to determine exactly one numerical value for the quantity.
     Example:

                P
                xº
           yº             zº
     Q                              R

     In ∆PQR, what is the value of x ?

     (1)    PQ = PR
     (2)    y = 40

     Explanation: According to statement (1) PQ = PR; therefore, ∆PQR is isosceles and y = z. Since x + y + z =
     180, it follows that x + 2y = 180. Since statement (1) does not give a value for y, you cannot answer the
     question using statement (1) alone. According to statement (2), y = 40; therefore, x + z = 140. Since
     statement (2) does not give a value for z, you cannot answer the question using statement (2) alone.
     Using both statements together, since x + 2y = 180 and the value of y is given, you can find the value
     of x. Therefore, BOTH statements (1) and (2) TOGETHER are sufficient to answer the question, but
     NEITHER statement ALONE is sufficient.
     Numbers: All numbers used are real numbers.
     Figures:
         • Figures conform to the information given in the question, but will not necessarily conform to the
             additional information given in statements (1) and (2).
         • Lines shown as straight are straight, and lines that appear jagged are also straight.
         • The positions of points, angles, regions, etc., exist in the order shown, and angle measures are
             greater than zero.
         • All figures lie in a plane unless otherwise indicated.




24
3.1 Diagnostic Test Quantitative Sample Questions




25.   If the units digit of integer n is greater than 2, what is   31.   Is the standard deviation of the set of measurements
      the units digit of n ?                                             x1, x2, x3, x4, . . ., x20 less than 3 ?

      (1)   The units digit of n is the same as the units digit          (1)   The variance for the set of measurements is 4.
            of n2.                                                       (2)   For each measurement, the difference between
      (2)   The units digit of n is the same as the units digit                the mean and that measurement is 2.
            of n3.
                                                                   32.   Is the range of the integers 6, 3, y, 4, 5, and x greater
26.   What is the value of the integer p ?                               than 9 ?
      (1)   Each of the integers 2, 3, and 5 is a factor of p.           (1)   y > 3x
      (2)   Each of the integers 2, 5, and 7 is a factor of p.           (2)   y>x>3

27.   If the length of Wanda’s telephone call was rounded up       33.   Is           ?
      to the nearest whole minute by her telephone
      company, then Wanda was charged for how many                       (1)   5x < 1
      minutes for her telephone call?                                    (2)   x<0
      (1)   The total charge for Wanda’s telephone call was
            $6.50.                                                 34.   Of the companies surveyed about the skills they
                                                                         required in prospective employees, 20 percent
      (2)   Wanda was charged $0.50 more for the first
                                                                         required both computer skills and writing skills. What
            minute of the telephone call than for each
                                                                         percent of the companies surveyed required neither
            minute after the first.
                                                                         computer skills nor writing skills?

28.   What is the perimeter of isosceles triangle MNP ?                  (1)   Of those companies surveyed that required
                                                                               computer skills, half required writing skills.
      (1)   MN = 16
                                                                         (2)   45 percent of the companies surveyed required
      (2)   NP = 20
                                                                               writing skills but not computer skills.

29.   In a survey of retailers, what percent had purchased
                                                                   35.   What is the value of w + q ?
      computers for business purposes?

      (1)   85 percent of the retailers surveyed who owned               (1)   3w = 3 − 3q
            their own store had purchased computers for                  (2)   5w + 5q = 5
            business purposes.
      (2)   40 percent of the retailers surveyed owned their       36.   If X and Y are points in a plane and X lies inside the
            own store.                                                   circle C with center O and radius 2, does Y lie inside
                                                                         circle C ?
30.   The only gift certificates that a certain store sold
      yesterday were worth either $100 each or $10 each. If              (1)   The length of line segment XY is 3.
      the store sold a total of 20 gift certificates yesterday,           (2)   The length of line segment OY is 1.5.
      how many gift certificates worth $10 each did the
      store sell yesterday?                                        37.   Is x > y ?

      (1)   The gift certificates sold by the store yesterday             (1)   x=y+2
            were worth a total of between $1,650 and
                                                                         (2)      =y−1
            $1,800.
      (2)   Yesterday the store sold more than 15 gift
            certificates worth $100 each.




                                                                                                                                  25
The Official Guide for GMAT® Review 12th Edition




38.   If Paula drove the distance from her home to her
      college at an average speed that was greater than            44.   If m and n are positive integers, is      an integer?
      70 kilometers per hour, did it take her less than
                                                                         (1)         is an integer.
      3 hours to drive this distance?

      (1)     The distance that Paula drove from her home to             (2)        is an integer.
              her college was greater than 200 kilometers.
                                                                   45.   Of the 66 people in a certain auditorium, at most
      (2)     The distance that Paula drove from her home to
                                                                         6 people have their birthdays in any one given month.
              her college was less than 205 kilometers.
                                                                         Does at least one person in the auditorium have a
                                                                         birthday in January?
39.   In the xy-plane, if line k has negative slope and passes
      through the point (−5,r ) , is the x-intercept of line k           (1)   More of the people in the auditorium have their
      positive?                                                                birthday in February than in March.

      (1)     The slope of line k is –5.                                 (2)   Five of the people in the auditorium have their
                                                                               birthday in March.
      (2)     r>0

                                                                   46.   Last year the average (arithmetic mean) salary of the
40.   If $5,000 invested for one year at p percent simple
                                                                         10 employees of Company X was $42,800. What is the
      annual interest yields $500, what amount must be
                                                                         average salary of the same 10 employees this year?
      invested at k percent simple annual interest for one
      year to yield the same number of dollars?                          (1)   For 8 of the 10 employees, this year’s salary is
                                                                               15 percent greater than last year’s salary.
      (1)     k = 0.8p
                                                                         (2)   For 2 of the 10 employees, this year’s salary is
      (2)     k=8
                                                                               the same as last year’s salary.
           x+y
41.   If       > 0, is x < 0 ?
            z                                                      47.   In a certain classroom, there are 80 books, of which
                                                                         24 are fiction and 23 are written in Spanish. How many
      (1)     x<y
                                                                         of the fiction books are written in Spanish?
      (2)     z<0
                                                                         (1)   Of the fiction books, there are 6 more that are
42.   Does the integer k have at least three different                         not written in Spanish than are written in
      positive prime factors?                                                  Spanish.
                                                                         (2)   Of the books written in Spanish, there are 5
               k                                                               more nonfiction books than fiction books.
      (1)        is an integer.
              15
               k                                                   48.   If p is the perimeter of rectangle Q, what is the value
      (2)        is an integer.
              10                                                         of p ?

43.   In City X last April, was the average (arithmetic mean)            (1)   Each diagonal of rectangle Q has length 10.
      daily high temperature greater than the median daily
                                                                         (2)   The area of rectangle Q is 48.
      high temperature?

      (1)     In City X last April, the sum of the 30 daily high
              temperatures was 2,160°.
      (2)     In City X last April, 60 percent of the daily high
              temperatures were less than the average daily
              high temperature.




26
3.2 Diagnostic Test Verbal Sample Questions




 3.2 Verbal Sample Questions
       Reading Comprehension
       Each of the reading comprehension questions is based on the content of a passage. After reading the
       passage, answer all questions pertaining to it on the basis of what is stated or implied in the passage. For
       each question, select the best answer of the choices given.




Line     According to economic signaling theory,                1.   Which of the following best describes the purpose of
         consumers may perceive the frequency with                   the sentence in lines 10–15 ?
         which an unfamiliar brand is advertised as a cue
         that the brand is of high quality. The notion that          (A)   To show that economic signaling theory fails to
(5)      highly advertised brands are associated with                      explain a finding
         high-quality products does have some empirical              (B)   To introduce a distinction not accounted for by
         support. Marquardt and McGann found that                          economic signaling theory
         heavily advertised products did indeed rank high            (C)   To account for an exception to a generalization
         on certain measures of product quality. Because                   suggested by Marquardt and McGann
(10)     large advertising expenditures represent
                                                                     (D)   To explain why Marquardt and McGann’s
         a significant investment on the part of a
                                                                           research was conducted
         manufacturer, only companies that expect to
                                                                     (E)   To offer an explanation for an observation
         recoup these costs in the long run, through
                                                                           reported by Marquardt and McGann
         consumers’ repeat purchases of the product,
(15)     can afford to spend such amounts.
            However, two studies by Kirmani have found          2.   The primary purpose of the passage is to
         that although consumers initially perceive expensive
                                                                     (A)   present findings that contradict one explanation
         advertising as a signal of high brand quality,
                                                                           for the effects of a particular advertising
         at some level of spending the manufacturer’s
                                                                           practice
(20)     advertising effort may be perceived as unreasonably
         high, implying low manufacturer confidence in                (B)   argue that theoretical explanations about the
         product quality. If consumers perceive excessive                  effects of a particular advertising practice are
         advertising effort as a sign of a manufacturer’s                  of limited value without empirical evidence
         desperation, the result may be less favorable               (C)   discuss how and why particular advertising
(25)     brand perceptions. In addition, a third study by                  practices may affect consumers’ perceptions
         Kirmani, of print advertisements, found that the            (D)   contrast the research methods used in two
         use of color affected consumer perception of                      different studies of a particular advertising
         brand quality. Because consumers recognize that                   practice
         color advertisements are more expensive than
                                                                     (E)   explain why a finding about consumer responses
(30)     black and white, the point at which repetition of an
                                                                           to a particular advertising practice was
         advertisement is perceived as excessive comes
                                                                           unexpected
         sooner for a color advertisement than for a black-
         and-white advertisement.




                                                                                                                              27
The Official Guide for GMAT® Review 12th Edition




3.   Kirmani’s research, as described in the passage,            5.   The passage suggests that Kirmani would be most
     suggests which of the following regarding consumers’             likely to agree with which of the following statements
     expectations about the quality of advertised products?           about consumers’ perceptions of the relationship
                                                                      between the frequency with which a product is
     (A)   Those expectations are likely to be highest if a           advertised and the product’s quality?
           manufacturer runs both black-and-white and
           color advertisements for the same product.                 (A)   Consumers’ perceptions about the frequency
     (B)   Those expectations can be shaped by the                          with which an advertisement appears are their
           presence of color in an advertisement as well as                 primary consideration when evaluating an
           by the frequency with which an advertisement                     advertisement’s claims about product quality.
           appears.                                                   (B)   Because most consumers do not notice the
     (C)   Those expectations are usually high for                          frequency of advertisement, it has little impact
           frequently advertised new brands but not for                     on most consumers’ expectations regarding
           frequently advertised familiar brands.                           product quality.

     (D)   Those expectations are likely to be higher for             (C)   Consumers perceive frequency of advertisement
           products whose black-and-white advertisements                    as a signal about product quality only when the
           are often repeated than for those whose color                    advertisement is for a product that is newly on
           advertisements are less often repeated.                          the market.

     (E)   Those expectations are less definitively shaped             (D)   The frequency of advertisement is not always
           by the manufacturer’s advertisements than by                     perceived by consumers to indicate that
           information that consumers gather from other                     manufacturers are highly confident about their
           sources.                                                         products’ quality.
                                                                      (E)   Consumers who try a new product that has been
4.   Kirmani’s third study, as described in the passage,                    frequently advertised are likely to perceive the
     suggests which of the following conclusions about a                    advertisement’s frequency as having been an
     black-and-white advertisement?                                         accurate indicator of the product’s quality.

     (A)   It can be repeated more frequently than a
           comparable color advertisement could before
           consumers begin to suspect low manufacturer
           confidence in the quality of the advertised
           product.
     (B)   It will have the greatest impact on consumers’
           perceptions of the quality of the advertised
           product if it appears during periods when a color
           version of the same advertisement is also being
           used.
     (C)   It will attract more attention from readers of the
           print publication in which it appears if it is used
           only a few times.
     (D)   It may be perceived by some consumers as
           more expensive than a comparable color
           advertisement.
     (E)   It is likely to be perceived by consumers as a
           sign of higher manufacturer confidence in the
           quality of the advertised product than a
           comparable color advertisement would be.




28
3.2 Diagnostic Test Verbal Sample Questions




Line   The idea of the brain as an information                 6.   The main purpose of the passage is to
       processor—a machine manipulating blips of energy
       according to fathomable rules—has come to                    (A)   propose an experiment
       dominate neuroscience. However, one enemy of                 (B)   analyze a function
(5)    the brain-as-computer metaphor is John R. Searle,            (C)   refute an argument
       a philosopher who argues that since computers
                                                                    (D)   explain a contradiction
       simply follow algorithms, they cannot deal with
                                                                    (E)   simulate a process
       important aspects of human thought such as
       meaning and content. Computers are syntactic,
(10)   rather than semantic, creatures. People, on the         7.   Which of the following is most consistent with Searle’s
       other hand, understand meaning because they have             reasoning as presented in the passage?
       something Searle obscurely calls the causal powers
                                                                    (A)   Meaning and content cannot be reduced to
       of the brain.
                                                                          algorithms.
           Yet how would a brain work if not by reducing
(15)   what it learns about the world to information—some           (B)   The process of digestion can be simulated
       kind of code that can be transmitted from neuron                   mechanically, but not on a computer.
       to neuron? What else could meaning and content               (C)   Simulated thoughts and real thoughts are
       be? If the code can be cracked, a computer should                  essentially similar because they are composed
       be able to simulate it, at least in principle. But                 primarily of information.
(20)   even if a computer could simulate the workings               (D)   A computer can use “causal powers” similar to
       of the mind, Searle would claim that the machine                   those of the human brain when processing
       would not really be thinking; it would just be acting              information.
       as if it were. His argument proceeds thus: if a
                                                                    (E)   Computer simulations of the world can achieve
       computer were used to simulate a stomach, with
                                                                          the complexity of the brain’s representations of
(25)   the stomach’s churnings faithfully reproduced on a
                                                                          the world.
       video screen, the machine would not be digesting
       real food. It would just be blindly manipulating the
                                                               8.   The author of the passage would be most likely to
       symbols that generate the visual display.
                                                                    agree with which of the following statements about the
           Suppose, though, that a stomach were simulated
                                                                    simulation of organ functions?
(30)   using plastic tubes, a motor to do the churning, a
       supply of digestive juices, and a timing mechanism.          (A)   An artificial device that achieves the functions of
       If food went in one end of the device, what came out               the stomach could be considered a valid model
       the other end would surely be digested food. Brains,               of the stomach.
       unlike stomachs, are information processors, and if
                                                                    (B)   Computer simulations of the brain are best used
(35)   one information processor were made to simulate
                                                                          to crack the brain’s codes of meaning and
       another information processor, it is hard to see
                                                                          content.
       how one and not the other could be said to think.
       Simulated thoughts and real thoughts are made of             (C)   Computer simulations of the brain challenge
       the same element: information. The representations                 ideas that are fundamental to psychology and
(40)   of the world that humans carry around in their heads               neuroscience.
       are already simulations. To accept Searle’s argument,        (D)   Because the brain and the stomach both act as
       one would have to deny the most fundamental notion                 processors, they can best be simulated by
       in psychology and neuroscience: that brains work                   mechanical devices.
       by processing information.                                   (E)   The computer’s limitations in simulating
                                                                          digestion suggest equal limitations in computer-
                                                                          simulated thinking.




                                                                                                                          29
The Official Guide for GMAT® Review 12th Edition




9.    It can be inferred that the author of the passage
      believes that Searle’s argument is flawed by its
      failure to

      (A)   distinguish between syntactic and semantic
            operations
      (B)   explain adequately how people, unlike
            computers, are able to understand meaning
      (C)   provide concrete examples illustrating its claims
            about thinking
      (D)   understand how computers use algorithms to
            process information
      (E)   decipher the code that is transmitted from
            neuron to neuron in the brain

10.   From the passage, it can be inferred that the author
      would agree with Searle on which of the following
      points?

      (A)   Computers operate by following algorithms.
      (B)   The human brain can never fully understand its
            own functions.
      (C)   The comparison of the brain to a machine is
            overly simplistic.
      (D)   The most accurate models of physical
            processes are computer simulations.
      (E)   Human thought and computer-simulated thought
            involve similar processes of representation.

11.   Which of the following most accurately represents
      Searle’s criticism of the brain-as-computer metaphor,
      as that criticism is described in the passage?

      (A)   The metaphor is not experimentally verifiable.
      (B)   The metaphor does not take into account the
            unique powers of the brain.
      (C)   The metaphor suggests that a brain’s functions
            can be simulated as easily as those of a
            stomach.
      (D)   The metaphor suggests that a computer can
            simulate the workings of the mind by using the
            codes of neural transmission.
      (E)   The metaphor is unhelpful because both the
            brain and the computer process information.




30
3.2 Diagnostic Test Verbal Sample Questions




Line   Women’s grassroots activism and their vision             12.   The primary purpose of the passage is to
       of a new civic consciousness lay at the heart of
       social reform in the United States throughout the              (A)   explain why women reformers of the Progressive
       Progressive Era, the period between the depression                   Era failed to achieve their goals
(5)    of 1893 and America’s entry into the Second                    (B)   discuss the origins of child labor laws in the late
       World War. Though largely disenfranchised except                     nineteenth and early twentieth centuries
       for school elections, white middle-class women                 (C)   compare the living conditions of working-class
       reformers won a variety of victories, notably in                     and middle-class women in the Progressive Era
       the improvement of working conditions, especially
                                                                      (D)   discuss an oversight on the part of women
(10)   for women and children. Ironically, though,
                                                                            reformers of the Progressive Era
       child labor legislation pitted women of different
                                                                      (E)   revise a traditional view of the role played by
       classes against one another. To the reformers,
                                                                            women reformers in enacting Progressive Era
       child labor and industrial home work were equally
                                                                            reforms
       inhumane practices that should be outlawed, but,
(15)   as a number of women historians have recently
       observed, working-class mothers did not always           13.   The view mentioned in line 17 of the passage refers to
       share this view. Given the precarious finances of               which of the following?
       working-class families and the necessity of pooling
                                                                      (A)   Some working-class mothers’ resistance to the
       the wages of as many family members as possible,
                                                                            enforcement of child labor laws
(20)   working-class families viewed the passage and
       enforcement of stringent child labor statutes as a             (B)   Reformers’ belief that child labor and industrial
       personal economic disaster and made strenuous                        home work should be abolished
       efforts to circumvent child labor laws. Yet                    (C)   Reformers’ opinions about how working-class
       reformers rarely understood this resistance in terms                 families raised their children
(25)   of the desperate economic situation of working-                (D)   Certain women historians’ observation that there
       class families, interpreting it instead as evidence                  was a lack of consensus between women of
       of poor parenting. This is not to dispute women                      different classes on the issue of child labor and
       reformers’ perception of child labor as a terribly                   industrial home work
       exploitative practice, but their understanding of
                                                                      (E)   Working-class families’ fears about the adverse
(30)   child labor and their legislative solutions for ending
                                                                            consequences that child labor laws would have
       it failed to take account of the economic needs of
                                                                            on their ability to earn an adequate living
       working-class families.

                                                                14.   The author of the passage mentions the observations
                                                                      of women historians (lines 15–17) most probably in
                                                                      order to

                                                                      (A)   provide support for an assertion made in the
                                                                            preceding sentence (lines 10–12)
                                                                      (B)   raise a question that is answered in the last
                                                                            sentence of the passage (lines 27–32)
                                                                      (C)   introduce an opinion that challenges a statement
                                                                            made in the first sentence of the passage
                                                                      (D)   offer an alternative view to the one attributed in
                                                                            the passage to working-class mothers
                                                                      (E)   point out a contradiction inherent in the
                                                                            traditional view of child labor reform as it is
                                                                            presented in the passage




                                                                                                                                31
The Official Guide for GMAT® Review 12th Edition




15.   The passage suggests that which of the following was       17.   According to the passage, one of the most striking
      a reason for the difference of opinion between                   achievements of white middle-class women reformers
      working-class mothers and women reformers on the                 during the Progressive Era was
      issue of child labor?
                                                                       (A)   gaining the right to vote in school elections
      (A)   Reformers’ belief that industrial home work was            (B)   mobilizing working-class women in the fight
            preferable to child labor outside the home
                                                                             against child labor
      (B)   Reformers’ belief that child labor laws should
            pertain to working conditions but not to pay               (C)   uniting women of different classes in grassroots
                                                                             activism
      (C)   Working-class mothers’ resentment at reformers’
            attempts to interfere with their parenting                 (D)   improving the economic conditions of working-
      (D)   Working-class mothers’ belief that child labor                   class families
            was an inhumane practice                                   (E)   improving women’s and children’s working
      (E)   Working-class families’ need for every                           conditions
            employable member of their families to earn
            money

16.   The author of the passage asserts which of the
      following about women reformers who tried to abolish
      child labor?

      (A)   They alienated working-class mothers by
            attempting to enlist them in agitating for
            progressive causes.
      (B)   They underestimated the prevalence of child
            labor among the working classes.
      (C)   They were correct in their conviction that child
            labor was deplorable but shortsighted about the
            impact of child labor legislation on working-class
            families.
      (D)   They were aggressive in their attempts to
            enforce child labor legislation, but were unable
            to prevent working-class families from
            circumventing them.
      (E)   They were prevented by their nearly total
            disenfranchisement from making significant
            progress in child labor reform.




32
GMAT Review 12th Edition
GMAT Review 12th Edition
GMAT Review 12th Edition
GMAT Review 12th Edition
GMAT Review 12th Edition
GMAT Review 12th Edition
GMAT Review 12th Edition
GMAT Review 12th Edition
GMAT Review 12th Edition
GMAT Review 12th Edition
GMAT Review 12th Edition
GMAT Review 12th Edition
GMAT Review 12th Edition
GMAT Review 12th Edition
GMAT Review 12th Edition
GMAT Review 12th Edition
GMAT Review 12th Edition
GMAT Review 12th Edition
GMAT Review 12th Edition
GMAT Review 12th Edition
GMAT Review 12th Edition
GMAT Review 12th Edition
GMAT Review 12th Edition
GMAT Review 12th Edition
GMAT Review 12th Edition
GMAT Review 12th Edition
GMAT Review 12th Edition
GMAT Review 12th Edition
GMAT Review 12th Edition
GMAT Review 12th Edition
GMAT Review 12th Edition
GMAT Review 12th Edition
GMAT Review 12th Edition
GMAT Review 12th Edition
GMAT Review 12th Edition
GMAT Review 12th Edition
GMAT Review 12th Edition
GMAT Review 12th Edition
GMAT Review 12th Edition
GMAT Review 12th Edition
GMAT Review 12th Edition
GMAT Review 12th Edition
GMAT Review 12th Edition
GMAT Review 12th Edition
GMAT Review 12th Edition
GMAT Review 12th Edition
GMAT Review 12th Edition
GMAT Review 12th Edition
GMAT Review 12th Edition
GMAT Review 12th Edition
GMAT Review 12th Edition
GMAT Review 12th Edition
GMAT Review 12th Edition
GMAT Review 12th Edition
GMAT Review 12th Edition
GMAT Review 12th Edition
GMAT Review 12th Edition
GMAT Review 12th Edition
GMAT Review 12th Edition
GMAT Review 12th Edition
GMAT Review 12th Edition
GMAT Review 12th Edition
GMAT Review 12th Edition
GMAT Review 12th Edition
GMAT Review 12th Edition
GMAT Review 12th Edition
GMAT Review 12th Edition
GMAT Review 12th Edition
GMAT Review 12th Edition
GMAT Review 12th Edition
GMAT Review 12th Edition
GMAT Review 12th Edition
GMAT Review 12th Edition
GMAT Review 12th Edition
GMAT Review 12th Edition
GMAT Review 12th Edition
GMAT Review 12th Edition
GMAT Review 12th Edition
GMAT Review 12th Edition
GMAT Review 12th Edition
GMAT Review 12th Edition
GMAT Review 12th Edition
GMAT Review 12th Edition
GMAT Review 12th Edition
GMAT Review 12th Edition
GMAT Review 12th Edition
GMAT Review 12th Edition
GMAT Review 12th Edition
GMAT Review 12th Edition
GMAT Review 12th Edition
GMAT Review 12th Edition
GMAT Review 12th Edition
GMAT Review 12th Edition
GMAT Review 12th Edition
GMAT Review 12th Edition
GMAT Review 12th Edition
GMAT Review 12th Edition
GMAT Review 12th Edition
GMAT Review 12th Edition
GMAT Review 12th Edition
GMAT Review 12th Edition
GMAT Review 12th Edition
GMAT Review 12th Edition
GMAT Review 12th Edition
GMAT Review 12th Edition
GMAT Review 12th Edition
GMAT Review 12th Edition
GMAT Review 12th Edition
GMAT Review 12th Edition
GMAT Review 12th Edition
GMAT Review 12th Edition
GMAT Review 12th Edition
GMAT Review 12th Edition
GMAT Review 12th Edition
GMAT Review 12th Edition
GMAT Review 12th Edition
GMAT Review 12th Edition
GMAT Review 12th Edition
GMAT Review 12th Edition
GMAT Review 12th Edition
GMAT Review 12th Edition
GMAT Review 12th Edition
GMAT Review 12th Edition
GMAT Review 12th Edition
GMAT Review 12th Edition
GMAT Review 12th Edition
GMAT Review 12th Edition
GMAT Review 12th Edition
GMAT Review 12th Edition
GMAT Review 12th Edition
GMAT Review 12th Edition
GMAT Review 12th Edition
GMAT Review 12th Edition
GMAT Review 12th Edition
GMAT Review 12th Edition
GMAT Review 12th Edition
GMAT Review 12th Edition
GMAT Review 12th Edition
GMAT Review 12th Edition
GMAT Review 12th Edition
GMAT Review 12th Edition
GMAT Review 12th Edition
GMAT Review 12th Edition
GMAT Review 12th Edition
GMAT Review 12th Edition
GMAT Review 12th Edition
GMAT Review 12th Edition
GMAT Review 12th Edition
GMAT Review 12th Edition
GMAT Review 12th Edition
GMAT Review 12th Edition
GMAT Review 12th Edition
GMAT Review 12th Edition
GMAT Review 12th Edition
GMAT Review 12th Edition
GMAT Review 12th Edition
GMAT Review 12th Edition
GMAT Review 12th Edition
GMAT Review 12th Edition
GMAT Review 12th Edition
GMAT Review 12th Edition
GMAT Review 12th Edition
GMAT Review 12th Edition
GMAT Review 12th Edition
GMAT Review 12th Edition
GMAT Review 12th Edition
GMAT Review 12th Edition
GMAT Review 12th Edition
GMAT Review 12th Edition
GMAT Review 12th Edition
GMAT Review 12th Edition
GMAT Review 12th Edition
GMAT Review 12th Edition
GMAT Review 12th Edition
GMAT Review 12th Edition
GMAT Review 12th Edition
GMAT Review 12th Edition
GMAT Review 12th Edition
GMAT Review 12th Edition
GMAT Review 12th Edition
GMAT Review 12th Edition
GMAT Review 12th Edition
GMAT Review 12th Edition
GMAT Review 12th Edition
GMAT Review 12th Edition
GMAT Review 12th Edition
GMAT Review 12th Edition
GMAT Review 12th Edition
GMAT Review 12th Edition
GMAT Review 12th Edition
GMAT Review 12th Edition
GMAT Review 12th Edition
GMAT Review 12th Edition
GMAT Review 12th Edition
GMAT Review 12th Edition
GMAT Review 12th Edition
GMAT Review 12th Edition
GMAT Review 12th Edition
GMAT Review 12th Edition
GMAT Review 12th Edition
GMAT Review 12th Edition
GMAT Review 12th Edition
GMAT Review 12th Edition
GMAT Review 12th Edition
GMAT Review 12th Edition
GMAT Review 12th Edition
GMAT Review 12th Edition
GMAT Review 12th Edition
GMAT Review 12th Edition
GMAT Review 12th Edition
GMAT Review 12th Edition
GMAT Review 12th Edition
GMAT Review 12th Edition
GMAT Review 12th Edition
GMAT Review 12th Edition
GMAT Review 12th Edition
GMAT Review 12th Edition
GMAT Review 12th Edition
GMAT Review 12th Edition
GMAT Review 12th Edition
GMAT Review 12th Edition
GMAT Review 12th Edition
GMAT Review 12th Edition
GMAT Review 12th Edition
GMAT Review 12th Edition
GMAT Review 12th Edition
GMAT Review 12th Edition
GMAT Review 12th Edition
GMAT Review 12th Edition
GMAT Review 12th Edition
GMAT Review 12th Edition
GMAT Review 12th Edition
GMAT Review 12th Edition
GMAT Review 12th Edition
GMAT Review 12th Edition
GMAT Review 12th Edition
GMAT Review 12th Edition
GMAT Review 12th Edition
GMAT Review 12th Edition
GMAT Review 12th Edition
GMAT Review 12th Edition
GMAT Review 12th Edition
GMAT Review 12th Edition
GMAT Review 12th Edition
GMAT Review 12th Edition
GMAT Review 12th Edition
GMAT Review 12th Edition
GMAT Review 12th Edition
GMAT Review 12th Edition
GMAT Review 12th Edition
GMAT Review 12th Edition
GMAT Review 12th Edition
GMAT Review 12th Edition
GMAT Review 12th Edition
GMAT Review 12th Edition
GMAT Review 12th Edition
GMAT Review 12th Edition
GMAT Review 12th Edition
GMAT Review 12th Edition
GMAT Review 12th Edition
GMAT Review 12th Edition
GMAT Review 12th Edition
GMAT Review 12th Edition
GMAT Review 12th Edition
GMAT Review 12th Edition
GMAT Review 12th Edition
GMAT Review 12th Edition
GMAT Review 12th Edition
GMAT Review 12th Edition
GMAT Review 12th Edition
GMAT Review 12th Edition
GMAT Review 12th Edition
GMAT Review 12th Edition
GMAT Review 12th Edition
GMAT Review 12th Edition
GMAT Review 12th Edition
GMAT Review 12th Edition
GMAT Review 12th Edition
GMAT Review 12th Edition
GMAT Review 12th Edition
GMAT Review 12th Edition
GMAT Review 12th Edition
GMAT Review 12th Edition
GMAT Review 12th Edition
GMAT Review 12th Edition
GMAT Review 12th Edition
GMAT Review 12th Edition
GMAT Review 12th Edition
GMAT Review 12th Edition
GMAT Review 12th Edition
GMAT Review 12th Edition
GMAT Review 12th Edition
GMAT Review 12th Edition
GMAT Review 12th Edition
GMAT Review 12th Edition
GMAT Review 12th Edition
GMAT Review 12th Edition
GMAT Review 12th Edition
GMAT Review 12th Edition
GMAT Review 12th Edition
GMAT Review 12th Edition
GMAT Review 12th Edition
GMAT Review 12th Edition
GMAT Review 12th Edition
GMAT Review 12th Edition
GMAT Review 12th Edition
GMAT Review 12th Edition
GMAT Review 12th Edition
GMAT Review 12th Edition
GMAT Review 12th Edition
GMAT Review 12th Edition
GMAT Review 12th Edition
GMAT Review 12th Edition
GMAT Review 12th Edition
GMAT Review 12th Edition
GMAT Review 12th Edition
GMAT Review 12th Edition
GMAT Review 12th Edition
GMAT Review 12th Edition
GMAT Review 12th Edition
GMAT Review 12th Edition
GMAT Review 12th Edition
GMAT Review 12th Edition
GMAT Review 12th Edition
GMAT Review 12th Edition
GMAT Review 12th Edition
GMAT Review 12th Edition
GMAT Review 12th Edition
GMAT Review 12th Edition
GMAT Review 12th Edition
GMAT Review 12th Edition
GMAT Review 12th Edition
GMAT Review 12th Edition
GMAT Review 12th Edition
GMAT Review 12th Edition
GMAT Review 12th Edition
GMAT Review 12th Edition
GMAT Review 12th Edition
GMAT Review 12th Edition
GMAT Review 12th Edition
GMAT Review 12th Edition
GMAT Review 12th Edition
GMAT Review 12th Edition
GMAT Review 12th Edition
GMAT Review 12th Edition
GMAT Review 12th Edition
GMAT Review 12th Edition
GMAT Review 12th Edition
GMAT Review 12th Edition
GMAT Review 12th Edition
GMAT Review 12th Edition
GMAT Review 12th Edition
GMAT Review 12th Edition
GMAT Review 12th Edition
GMAT Review 12th Edition
GMAT Review 12th Edition
GMAT Review 12th Edition
GMAT Review 12th Edition
GMAT Review 12th Edition
GMAT Review 12th Edition
GMAT Review 12th Edition
GMAT Review 12th Edition
GMAT Review 12th Edition
GMAT Review 12th Edition
GMAT Review 12th Edition
GMAT Review 12th Edition
GMAT Review 12th Edition
GMAT Review 12th Edition
GMAT Review 12th Edition
GMAT Review 12th Edition
GMAT Review 12th Edition
GMAT Review 12th Edition
GMAT Review 12th Edition
GMAT Review 12th Edition
GMAT Review 12th Edition
GMAT Review 12th Edition
GMAT Review 12th Edition
GMAT Review 12th Edition
GMAT Review 12th Edition
GMAT Review 12th Edition
GMAT Review 12th Edition
GMAT Review 12th Edition
GMAT Review 12th Edition
GMAT Review 12th Edition
GMAT Review 12th Edition
GMAT Review 12th Edition
GMAT Review 12th Edition
GMAT Review 12th Edition
GMAT Review 12th Edition
GMAT Review 12th Edition
GMAT Review 12th Edition
GMAT Review 12th Edition
GMAT Review 12th Edition
GMAT Review 12th Edition
GMAT Review 12th Edition
GMAT Review 12th Edition
GMAT Review 12th Edition
GMAT Review 12th Edition
GMAT Review 12th Edition
GMAT Review 12th Edition
GMAT Review 12th Edition
GMAT Review 12th Edition
GMAT Review 12th Edition
GMAT Review 12th Edition
GMAT Review 12th Edition
GMAT Review 12th Edition
GMAT Review 12th Edition
GMAT Review 12th Edition
GMAT Review 12th Edition
GMAT Review 12th Edition
GMAT Review 12th Edition
GMAT Review 12th Edition
GMAT Review 12th Edition
GMAT Review 12th Edition
GMAT Review 12th Edition
GMAT Review 12th Edition
GMAT Review 12th Edition
GMAT Review 12th Edition
GMAT Review 12th Edition
GMAT Review 12th Edition
GMAT Review 12th Edition
GMAT Review 12th Edition
GMAT Review 12th Edition
GMAT Review 12th Edition
GMAT Review 12th Edition
GMAT Review 12th Edition
GMAT Review 12th Edition
GMAT Review 12th Edition
GMAT Review 12th Edition
GMAT Review 12th Edition
GMAT Review 12th Edition
GMAT Review 12th Edition
GMAT Review 12th Edition
GMAT Review 12th Edition
GMAT Review 12th Edition
GMAT Review 12th Edition
GMAT Review 12th Edition
GMAT Review 12th Edition
GMAT Review 12th Edition
GMAT Review 12th Edition
GMAT Review 12th Edition
GMAT Review 12th Edition
GMAT Review 12th Edition
GMAT Review 12th Edition
GMAT Review 12th Edition
GMAT Review 12th Edition
GMAT Review 12th Edition
GMAT Review 12th Edition
GMAT Review 12th Edition
GMAT Review 12th Edition
GMAT Review 12th Edition
GMAT Review 12th Edition
GMAT Review 12th Edition
GMAT Review 12th Edition
GMAT Review 12th Edition
GMAT Review 12th Edition
GMAT Review 12th Edition
GMAT Review 12th Edition
GMAT Review 12th Edition
GMAT Review 12th Edition
GMAT Review 12th Edition
GMAT Review 12th Edition
GMAT Review 12th Edition
GMAT Review 12th Edition
GMAT Review 12th Edition
GMAT Review 12th Edition
GMAT Review 12th Edition
GMAT Review 12th Edition
GMAT Review 12th Edition
GMAT Review 12th Edition
GMAT Review 12th Edition
GMAT Review 12th Edition
GMAT Review 12th Edition
GMAT Review 12th Edition
GMAT Review 12th Edition
GMAT Review 12th Edition
GMAT Review 12th Edition
GMAT Review 12th Edition
GMAT Review 12th Edition
GMAT Review 12th Edition
GMAT Review 12th Edition
GMAT Review 12th Edition
GMAT Review 12th Edition
GMAT Review 12th Edition
GMAT Review 12th Edition
GMAT Review 12th Edition
GMAT Review 12th Edition
GMAT Review 12th Edition
GMAT Review 12th Edition
GMAT Review 12th Edition
GMAT Review 12th Edition
GMAT Review 12th Edition
GMAT Review 12th Edition
GMAT Review 12th Edition
GMAT Review 12th Edition
GMAT Review 12th Edition
GMAT Review 12th Edition
GMAT Review 12th Edition
GMAT Review 12th Edition
GMAT Review 12th Edition
GMAT Review 12th Edition
GMAT Review 12th Edition
GMAT Review 12th Edition
GMAT Review 12th Edition
GMAT Review 12th Edition
GMAT Review 12th Edition
GMAT Review 12th Edition
GMAT Review 12th Edition
GMAT Review 12th Edition
GMAT Review 12th Edition
GMAT Review 12th Edition
GMAT Review 12th Edition
GMAT Review 12th Edition
GMAT Review 12th Edition
GMAT Review 12th Edition
GMAT Review 12th Edition
GMAT Review 12th Edition
GMAT Review 12th Edition
GMAT Review 12th Edition
GMAT Review 12th Edition
GMAT Review 12th Edition
GMAT Review 12th Edition
GMAT Review 12th Edition
GMAT Review 12th Edition
GMAT Review 12th Edition
GMAT Review 12th Edition
GMAT Review 12th Edition
GMAT Review 12th Edition
GMAT Review 12th Edition
GMAT Review 12th Edition
GMAT Review 12th Edition
GMAT Review 12th Edition
GMAT Review 12th Edition
GMAT Review 12th Edition
GMAT Review 12th Edition
GMAT Review 12th Edition
GMAT Review 12th Edition
GMAT Review 12th Edition
GMAT Review 12th Edition
GMAT Review 12th Edition
GMAT Review 12th Edition
GMAT Review 12th Edition
GMAT Review 12th Edition
GMAT Review 12th Edition
GMAT Review 12th Edition
GMAT Review 12th Edition
GMAT Review 12th Edition
GMAT Review 12th Edition
GMAT Review 12th Edition
GMAT Review 12th Edition
GMAT Review 12th Edition
GMAT Review 12th Edition
GMAT Review 12th Edition
GMAT Review 12th Edition
GMAT Review 12th Edition
GMAT Review 12th Edition
GMAT Review 12th Edition
GMAT Review 12th Edition
GMAT Review 12th Edition
GMAT Review 12th Edition
GMAT Review 12th Edition
GMAT Review 12th Edition
GMAT Review 12th Edition
GMAT Review 12th Edition
GMAT Review 12th Edition
GMAT Review 12th Edition
GMAT Review 12th Edition
GMAT Review 12th Edition
GMAT Review 12th Edition
GMAT Review 12th Edition
GMAT Review 12th Edition
GMAT Review 12th Edition
GMAT Review 12th Edition
GMAT Review 12th Edition
GMAT Review 12th Edition
GMAT Review 12th Edition
GMAT Review 12th Edition
GMAT Review 12th Edition
GMAT Review 12th Edition
GMAT Review 12th Edition
GMAT Review 12th Edition
GMAT Review 12th Edition
GMAT Review 12th Edition
GMAT Review 12th Edition
GMAT Review 12th Edition
GMAT Review 12th Edition
GMAT Review 12th Edition
GMAT Review 12th Edition
GMAT Review 12th Edition
GMAT Review 12th Edition
GMAT Review 12th Edition
GMAT Review 12th Edition
GMAT Review 12th Edition
GMAT Review 12th Edition
GMAT Review 12th Edition
GMAT Review 12th Edition
GMAT Review 12th Edition
GMAT Review 12th Edition
GMAT Review 12th Edition
GMAT Review 12th Edition
GMAT Review 12th Edition
GMAT Review 12th Edition
GMAT Review 12th Edition
GMAT Review 12th Edition
GMAT Review 12th Edition
GMAT Review 12th Edition
GMAT Review 12th Edition
GMAT Review 12th Edition
GMAT Review 12th Edition
GMAT Review 12th Edition
GMAT Review 12th Edition
GMAT Review 12th Edition
GMAT Review 12th Edition
GMAT Review 12th Edition
GMAT Review 12th Edition
GMAT Review 12th Edition
GMAT Review 12th Edition
GMAT Review 12th Edition
GMAT Review 12th Edition
GMAT Review 12th Edition
GMAT Review 12th Edition
GMAT Review 12th Edition
GMAT Review 12th Edition
GMAT Review 12th Edition
GMAT Review 12th Edition
GMAT Review 12th Edition
GMAT Review 12th Edition
GMAT Review 12th Edition
GMAT Review 12th Edition
GMAT Review 12th Edition
GMAT Review 12th Edition
GMAT Review 12th Edition
GMAT Review 12th Edition
GMAT Review 12th Edition
GMAT Review 12th Edition
GMAT Review 12th Edition
GMAT Review 12th Edition
GMAT Review 12th Edition
GMAT Review 12th Edition
GMAT Review 12th Edition
GMAT Review 12th Edition
GMAT Review 12th Edition
GMAT Review 12th Edition
GMAT Review 12th Edition
GMAT Review 12th Edition
GMAT Review 12th Edition
GMAT Review 12th Edition
GMAT Review 12th Edition
GMAT Review 12th Edition
GMAT Review 12th Edition
GMAT Review 12th Edition
GMAT Review 12th Edition
GMAT Review 12th Edition
GMAT Review 12th Edition
GMAT Review 12th Edition
GMAT Review 12th Edition
GMAT Review 12th Edition
GMAT Review 12th Edition
GMAT Review 12th Edition
GMAT Review 12th Edition
GMAT Review 12th Edition
GMAT Review 12th Edition
GMAT Review 12th Edition
GMAT Review 12th Edition
GMAT Review 12th Edition
GMAT Review 12th Edition
GMAT Review 12th Edition
GMAT Review 12th Edition
GMAT Review 12th Edition
GMAT Review 12th Edition
GMAT Review 12th Edition
GMAT Review 12th Edition
GMAT Review 12th Edition
GMAT Review 12th Edition
GMAT Review 12th Edition
GMAT Review 12th Edition
GMAT Review 12th Edition
GMAT Review 12th Edition
GMAT Review 12th Edition
GMAT Review 12th Edition
GMAT Review 12th Edition
GMAT Review 12th Edition
GMAT Review 12th Edition
GMAT Review 12th Edition
GMAT Review 12th Edition
GMAT Review 12th Edition
GMAT Review 12th Edition
GMAT Review 12th Edition
GMAT Review 12th Edition
GMAT Review 12th Edition
GMAT Review 12th Edition
GMAT Review 12th Edition
GMAT Review 12th Edition
GMAT Review 12th Edition
GMAT Review 12th Edition
GMAT Review 12th Edition
GMAT Review 12th Edition
GMAT Review 12th Edition
GMAT Review 12th Edition
GMAT Review 12th Edition
GMAT Review 12th Edition
GMAT Review 12th Edition
GMAT Review 12th Edition
GMAT Review 12th Edition
GMAT Review 12th Edition
GMAT Review 12th Edition
GMAT Review 12th Edition
GMAT Review 12th Edition
GMAT Review 12th Edition
GMAT Review 12th Edition
GMAT Review 12th Edition
GMAT Review 12th Edition
GMAT Review 12th Edition
GMAT Review 12th Edition
GMAT Review 12th Edition
GMAT Review 12th Edition
GMAT Review 12th Edition
GMAT Review 12th Edition
GMAT Review 12th Edition
GMAT Review 12th Edition
GMAT Review 12th Edition
GMAT Review 12th Edition
GMAT Review 12th Edition
GMAT Review 12th Edition
GMAT Review 12th Edition
GMAT Review 12th Edition
GMAT Review 12th Edition
GMAT Review 12th Edition
GMAT Review 12th Edition
GMAT Review 12th Edition
GMAT Review 12th Edition
GMAT Review 12th Edition
GMAT Review 12th Edition
GMAT Review 12th Edition
GMAT Review 12th Edition
GMAT Review 12th Edition
GMAT Review 12th Edition
GMAT Review 12th Edition
GMAT Review 12th Edition
GMAT Review 12th Edition
GMAT Review 12th Edition
GMAT Review 12th Edition
GMAT Review 12th Edition
GMAT Review 12th Edition
GMAT Review 12th Edition
GMAT Review 12th Edition
GMAT Review 12th Edition
GMAT Review 12th Edition
GMAT Review 12th Edition
GMAT Review 12th Edition
GMAT Review 12th Edition
GMAT Review 12th Edition
GMAT Review 12th Edition
GMAT Review 12th Edition
GMAT Review 12th Edition
GMAT Review 12th Edition
GMAT Review 12th Edition
GMAT Review 12th Edition
GMAT Review 12th Edition
GMAT Review 12th Edition
GMAT Review 12th Edition
GMAT Review 12th Edition
GMAT Review 12th Edition
GMAT Review 12th Edition
GMAT Review 12th Edition
GMAT Review 12th Edition
GMAT Review 12th Edition
GMAT Review 12th Edition
GMAT Review 12th Edition
GMAT Review 12th Edition
GMAT Review 12th Edition
GMAT Review 12th Edition
GMAT Review 12th Edition
GMAT Review 12th Edition
GMAT Review 12th Edition
GMAT Review 12th Edition
GMAT Review 12th Edition
GMAT Review 12th Edition
GMAT Review 12th Edition
GMAT Review 12th Edition
GMAT Review 12th Edition
GMAT Review 12th Edition
GMAT Review 12th Edition
GMAT Review 12th Edition
GMAT Review 12th Edition
GMAT Review 12th Edition
GMAT Review 12th Edition
GMAT Review 12th Edition
GMAT Review 12th Edition
GMAT Review 12th Edition
GMAT Review 12th Edition
GMAT Review 12th Edition
GMAT Review 12th Edition
GMAT Review 12th Edition
GMAT Review 12th Edition
GMAT Review 12th Edition
GMAT Review 12th Edition
GMAT Review 12th Edition
GMAT Review 12th Edition
GMAT Review 12th Edition
GMAT Review 12th Edition
GMAT Review 12th Edition
GMAT Review 12th Edition
GMAT Review 12th Edition
GMAT Review 12th Edition
GMAT Review 12th Edition
GMAT Review 12th Edition
GMAT Review 12th Edition
GMAT Review 12th Edition

More Related Content

Similar to GMAT Review 12th Edition

gmat-exam-success.pdf
gmat-exam-success.pdfgmat-exam-success.pdf
gmat-exam-success.pdfSumni Uchiha
 
Gmat test hub intro
Gmat test hub introGmat test hub intro
Gmat test hub introWordpandit
 
Ace the MBA Race
Ace the MBA RaceAce the MBA Race
Ace the MBA RaceCrackVerbal
 
Test strategy &-testplanning
Test strategy &-testplanningTest strategy &-testplanning
Test strategy &-testplanningsrivinayak
 
Cat to gmat chennai
Cat to gmat   chennaiCat to gmat   chennai
Cat to gmat chennaiCrackVerbal
 
Aqa specification
Aqa specificationAqa specification
Aqa specificationsaltashict
 
My Experiments In Agile Testing in Yahoo.pptx
My Experiments In Agile Testing in Yahoo.pptxMy Experiments In Agile Testing in Yahoo.pptx
My Experiments In Agile Testing in Yahoo.pptxBaiju Joseph
 
Agile testing experiments
Agile testing experimentsAgile testing experiments
Agile testing experimentsBaiju Joseph
 
Bangalore cat to gmat -
Bangalore   cat to gmat -Bangalore   cat to gmat -
Bangalore cat to gmat -CrackVerbal
 
PMP - What PMP expects you to prepare...
PMP - What PMP expects you to prepare...PMP - What PMP expects you to prepare...
PMP - What PMP expects you to prepare...sandeep sonkusale
 
Quality improvement for documentation case study 20120419
Quality improvement for documentation case study 20120419Quality improvement for documentation case study 20120419
Quality improvement for documentation case study 20120419Louise Tincher
 
Anton Muzhailo - Practical Test Process Improvement using ISTQB
Anton Muzhailo - Practical Test Process Improvement using ISTQBAnton Muzhailo - Practical Test Process Improvement using ISTQB
Anton Muzhailo - Practical Test Process Improvement using ISTQBIevgenii Katsan
 
Qa mock up interview for manual testing
Qa mock up interview for manual testingQa mock up interview for manual testing
Qa mock up interview for manual testingKadharBashaJ
 
Introducing QA Into an Agile Environment
Introducing QA Into an Agile EnvironmentIntroducing QA Into an Agile Environment
Introducing QA Into an Agile EnvironmentJoseph Beale
 

Similar to GMAT Review 12th Edition (20)

gmat-exam-success.pdf
gmat-exam-success.pdfgmat-exam-success.pdf
gmat-exam-success.pdf
 
Gmat
GmatGmat
Gmat
 
Gmat test hub intro
Gmat test hub introGmat test hub intro
Gmat test hub intro
 
Atdd half day_new_1_up
Atdd half day_new_1_upAtdd half day_new_1_up
Atdd half day_new_1_up
 
Ace the MBA Race
Ace the MBA RaceAce the MBA Race
Ace the MBA Race
 
Test strategy &-testplanning
Test strategy &-testplanningTest strategy &-testplanning
Test strategy &-testplanning
 
Cat to gmat chennai
Cat to gmat   chennaiCat to gmat   chennai
Cat to gmat chennai
 
Aqa specification
Aqa specificationAqa specification
Aqa specification
 
My Experiments In Agile Testing in Yahoo.pptx
My Experiments In Agile Testing in Yahoo.pptxMy Experiments In Agile Testing in Yahoo.pptx
My Experiments In Agile Testing in Yahoo.pptx
 
Gmat secrets
Gmat secretsGmat secrets
Gmat secrets
 
Agile testing experiments
Agile testing experimentsAgile testing experiments
Agile testing experiments
 
Bangalore cat to gmat -
Bangalore   cat to gmat -Bangalore   cat to gmat -
Bangalore cat to gmat -
 
PMP - What PMP expects you to prepare...
PMP - What PMP expects you to prepare...PMP - What PMP expects you to prepare...
PMP - What PMP expects you to prepare...
 
Quality improvement for documentation case study 20120419
Quality improvement for documentation case study 20120419Quality improvement for documentation case study 20120419
Quality improvement for documentation case study 20120419
 
Anton Muzhailo - Practical Test Process Improvement using ISTQB
Anton Muzhailo - Practical Test Process Improvement using ISTQBAnton Muzhailo - Practical Test Process Improvement using ISTQB
Anton Muzhailo - Practical Test Process Improvement using ISTQB
 
GMAT Tips to score 700
GMAT Tips to score 700GMAT Tips to score 700
GMAT Tips to score 700
 
Gcse business specs
Gcse business specsGcse business specs
Gcse business specs
 
Qa mock up interview for manual testing
Qa mock up interview for manual testingQa mock up interview for manual testing
Qa mock up interview for manual testing
 
Introducing QA Into an Agile Environment
Introducing QA Into an Agile EnvironmentIntroducing QA Into an Agile Environment
Introducing QA Into an Agile Environment
 
Baf Specification
Baf SpecificationBaf Specification
Baf Specification
 

GMAT Review 12th Edition

  • 1. more than 1 million copies sold worldwide ® 12th EDITION GMAT REVIEW The only study guide with more than 800 past GMAT ® questions—and their answers— —by the creators of the test. ~The ~ L ICIA OFF uide THE OFFICIAL GUIDE FOR G GMAT ® REVIEW, 12TH EDITION • Actual questions from past GMAT tests • Diagnostic section helps you assess where to focus your test-prep efforts • Insights into the GMAT exam that debunk test-taking myths ® From the Graduate Management Admission Council
  • 2.
  • 3. more than 1 million copies sold worldwide ® 12th EDITION GMAT REVIEW The only study guide with more than 800 past GMAT ® questions—and their answers— —by the creators of the test. ~The ~ L ICIA OFF uide THE OFFICIAL GUIDE FOR G GMAT ® REVIEW, 12TH EDITION • Actual questions from past GMAT tests • Diagnostic section helps you assess where to focus your test-prep efforts • Insights into the GMAT exam that debunk test-taking myths ® From the Graduate Management Admission Council
  • 4. THE OFFICIAL GUIDE FOR GMAT® REVIEW, 12TH EDITION Copyright © 2009 by the Graduate Management Admission Council®. All rights reserved. Published by Wiley Publishing, Inc., Hoboken, New Jersey No part of this publication may be reproduced, stored in a retrieval system or transmitted in any form or by any means, electronic, mechanical, photocopying, recording, scanning or otherwise, except as permitted under Sections 107 or 108 of the 1976 United States Copyright Act, without either the prior written permission of the Publisher, or authorization through payment of the appropriate per-copy fee to the Copyright Clearance Center, 222 Rosewood Drive, Danvers, MA 01923, (978) 750-8400, fax (978) 646-8600, or on the web at www.copyright.com. Requests to the Publisher for permission should be addressed to the Permissions Department, John Wiley & Sons, Inc., 111 River Street, Hoboken, NJ 07030, (201) 748-6011, fax (201) 748-6008, or online at http://www.wiley.com/go/permissions. The publisher and the author make no representations or warranties with respect to the accuracy or completeness of the contents of this work and specifically disclaim all warranties, including without limitation warranties of fitness for a particular purpose. No warranty may be created or extended by sales or promotional materials. The advice and strategies contained herein may not be suitable for every situation. This work is sold with the understanding that the publisher is not engaged in rendering legal, accounting, or other professional services. If professional assistance is required, the services of a competent professional person should be sought. Neither the publisher nor the author shall be liable for damages arising here from. The fact that an organization or Web site is referred to in this work as a citation and/or a potential source of further information does not mean that the author or the publisher endorses the information the organization or Web site may provide or recommendations it may make. Further, readers should be aware that Internet Web sites listed in this work may have changed or disappeared between when this work was written and when it is read. Trademarks: Wiley, the Wiley Publishing logo, and related trademarks are trademarks or registered trademarks of John Wiley & Sons, Inc. and/or its affiliates. GMAC®, GMAT®, GMAT CAT®, Graduate Management Admission Council®, and Graduate Management Admission Test® are registered trademarks of the Graduate Management Admission Council® (GMAC®). Creating Access to Graduate Business Educationsm is a service mark of the Graduate Management Admission Council®. All other trademarks are the property of their respective owners. Wiley Publishing, Inc. is not associated with any product or vendor mentioned in this book. For general information on our other products and services or to obtain technical support please contact our Customer Care Department within the U.S. at (877) 762-2974, outside the U.S. at (317) 572-3993 or fax (317) 572-4002. Wiley also publishes its books in a variety of electronic formats. Some content that appears in print may not be available in electronic books. For more information about Wiley products, please visit our Web site at www.wiley.com. Library of Congress Control Number: 2008940699 ISBN: 978-0-470-44974-5 Printed in the United States of America 10 9 8 7 6 5 4 3 2 1 Book production by Wiley Publishing, Inc. Composition Services Charles Forster, Designer Mike Wilson, Production Designer
  • 5. Table of Contents 1.0 What Is the GMAT®? 6 1.1 Why Take the GMAT® Test? 7 1.2 GMAT® Test Format 8 1.3 What Is the Content of the Test Like? 10 1.4 Quantitative Section 10 1.5 Verbal Section 10 1.6 What Computer Skills Will I Need? 11 1.7 What Are the Test Centers Like? 11 1.8 How Are Scores Calculated? 11 1.9 Analytical Writing Assessment Scores 12 1.10 Test Development Process 13 2.0 How to Prepare 14 2.1 How Can I Best Prepare to Take the Test? 15 2.2 What About Practice Tests? 15 2.3 How Should I Use the Diagnostic Test? 16 2.4 Where Can I Get Additional Practice? 16 2.5 General Test-Taking Suggestions 16 3.0 Diagnostic Test 18 3.1 Quantitative Sample Questions 20 3.2 Verbal Sample Questions 27 3.3 Quantitative and Verbal Answer Keys 45 3.4 Interpretive Guide 45 3.5 Quantitative Answer Explanations 46 3.6 Verbal Answer Explanations 65 4.0 Math Review 106 4.1 Arithmetic 108 4.2 Algebra 120 4.3 Geometry 127 4.4 Word Problems 140 5.0 Problem Solving 148 5.1 Test-Taking Strategies 150 5.2 The Directions 150 5.3 Sample Questions 152 5.4 Answer Key 186 5.5 Answer Explanations 188 6.0 Data Sufficiency 266 6.1 Test-Taking Strategies 268 6.2 The Directions 270 6.3 Sample Questions 272 6.4 Answer Key 289 6.5 Answer Explanations 290 7.0 Reading Comprehension 352 7.1 What Is Measured 354 7.2 Test-Taking Strategies 356 7.3 The Directions 357 7.4 Sample Questions 358 7.5 Answer Key 408 7.6 Answer Explanations 409
  • 6. 8.0 Critical Reasoning 482 8.1 What Is Measured 484 8.2 Test-Taking Strategies 484 8.3 The Directions 485 8.4 Sample Questions 486 8.5 Answer Key 525 8.6 Answer Explanations 526 9.0 Sentence Correction 650 9.1 Basic English Grammar Rules 651 9.2 Study Suggestions 656 9.3 What Is Measured 656 9.4 Test-Taking Strategies 656 9.5 The Directions 657 9.6 Sample Questions 658 9.7 Answer Key 684 9.8 Answer Explanations 685 10.0 Analytical Writing Assessment 758 10.1 What Is Measured 759 10.2 Test-Taking Strategies 760 10.3 The Directions 761 10.4 GMAT® Scoring Guide: Analysis of an Issue 762 10.5 Sample: Analysis of an Issue 764 10.6 Analysis of an Issue Sample Topics 769 10.7 GMAT® Scoring Guide: Analysis of an Argument 790 10.8 Sample: Analysis of an Argument 792 10.9 Analysis of an Argument Sample Topics 796 Appendix A Percentile Ranking Tables 828 Appendix B Answer Sheets 834 Diagnostic Answer Sheet 835 Problem Solving Answer Sheet 836 Data Sufficiency Answer Sheet 837 Reading Comprehension Answer Sheet 838 Critical Reasoning Answer Sheet 839 Sentence Correction Answer Sheet 840
  • 7. Dear Future Business Leader, By using this book to prepare for the GMAT® test, you are taking a very important step toward gaining admission to a high-quality business or management program and achieving a rewarding career in management. I applaud your decision. The Graduate Management Admission Council® developed the GMAT test more than 50 years ago to help leading graduate schools of business and management choose the applicants who best suit their programs. Today, the test is used by more than 1,800 graduate programs and is given to test takers daily in more than 110 countries around the world. Programs that use GMAT scores in selective admissions have helped establish the MBA degree as a hallmark of excellence worldwide. Why do GMAT scores matter so much? Other admissions factors—such as work experience, grades, admissions essays, and interviews—can say something about who you are and what you have done in your career, but only your GMAT scores can tell schools how you are likely to perform academically in the business school courses that are fundamental to the MBA degree. In fact, the test has been proven reliable as a predictor of academic performance for more than half a century. In other words, business schools that require you to take the GMAT really care about the quality of their student body. And excellent MBA students mean a stronger MBA program, a more enriching learning environment, and a more valuable degree for you to take into the business world. By enrolling in a school that uses the GMAT test for your graduate business degree, you will maximize the value of your degree, and that value will pay off in many ways, throughout your career. I wish you great success in preparing for this important next step in your professional education, and I wish you a very rewarding management career. Sincerely David A. Wilson President and CEO Graduate Management Admission Council®
  • 8. 1.0 What Is the GMAT®? 6
  • 9. 1.0 What Is the GMAT®? 1.0 What Is the GMAT®? The Graduate Management Admission Test® (GMAT®) is a standardized, three-part test delivered in English. The test was designed to help admissions officers evaluate how suitable individual applicants are for their graduate business and management programs. It measures basic verbal, mathematical, and analytical writing skills that a test taker has developed over a long period of time through education and work. The GMAT test does not a measure a person’s knowledge of specific fields of study. Graduate business and management programs enroll people from many different undergraduate and work backgrounds, so rather than test your mastery of any particular subject area, the GMAT test will assess your acquired skills. Your GMAT score will give admissions officers a statistically reliable measure of how well you are likely to perform academically in the core curriculum of a graduate business program. Of course, there are many other qualifications that can help people succeed in business school and in their careers—for instance, job experience, leadership ability, motivation, and interpersonal skills. The GMAT test does not gauge these qualities. That is why your GMAT score is intended to be used as one standard admissions criterion among other, more subjective, criteria, such as admissions essays and interviews. 1.1 Why Take the GMAT® Test? GMAT scores are used by admissions officers in roughly 1,800 graduate business and management programs worldwide. Schools that require prospective students to submit GMAT scores in the application process are generally interested in admitting the best-qualified applicants for their programs, which means that you may find a more -vs- FACT beneficial learning environment at schools that require GMAT scores as part of your application. – If I don’t score in the 90th percentile, I won’t get into Because the GMAT test gauges skills that are any school I choose. important to successful study of business and F – Very few people get very management at the graduate level, your scores will high scores. give you a good indication of how well prepared you are to succeed academically in a graduate Fewer than 50 of the more than 200,000 management program; how well you do on the test people taking the GMAT test each year get a perfect score of 800. Thus, while may also help you choose the business schools to you may be exceptionally capable, the odds which you apply. Furthermore, the percentile table are against your achieving a perfect score. you receive with your scores will tell you how your Also, the GMAT test is just one piece of your application packet. Admissions officers performance on the test compares to the use GMAT scores in conjunction with performance of other test takers, giving you one undergraduate records, application essays, way to gauge your competition for admission to interviews, letters of recommendation, and other information when deciding whom to business school. accept into their programs. 7
  • 10. The Official Guide for GMAT® Review 12th Edition Schools consider many different aspects of an application before making an admissions decision, so even if you score well on the GMAT test, you should contact the schools that interest you to learn more about them and to ask about how they use GMAT scores and other admissions criteria (such as your undergraduate grades, essays, and letters of recommendation) to evaluate candidates for admission. School admissions offices, school Web sites, and materials published by the school are the best sources for you to tap when you are doing research about where you might want to go to business school. For more information about how schools should use GMAT scores in admissions decisions, please read Appendix A of this book. For more information on the GMAT, registering to take the test, sending your scores to schools, and applying to business school, please visit our Web site at www.mba.com. 1.2 GMAT® Test Format The GMAT test consists of four separately timed sections (see the table on the next page). You start the test with two 30-minute Analytical Writing Assessment (AWA) questions that require you to type your responses using the computer keyboard. The writing section is followed by two 75-minute, multiple-choice sections: the Quantitative and Verbal sections of the test. The GMAT is a computer-adaptive test (CAT), which means that in the multiple-choice sections -vs- FACT of the test, the computer constantly gauges how well you are doing on the test and presents you – Getting an easier question with questions that are appropriate to your ability means I answered the last one level. These questions are drawn from a huge pool wrong. of possible test questions. So, although we talk about the GMAT as one test, the GMAT test you F – Getting an easier question take may be completely different from the test of does not necessarily mean you got the previous question the person sitting next to you. wrong. Here’s how it works. At the start of each GMAT To ensure that everyone receives the same multiple-choice section (Verbal and Quantitative), content, the test selects a specific number of questions of each type. The test may call you will be presented with a question of moderate for your next question to be a relatively difficulty. The computer uses your response to that hard problem-solving item involving first question to determine which question to arithmetic operations. But, if there are no more relatively difficult problem-solving present next. If you respond correctly, the test items involving arithmetic, you might be usually will give you questions of increasing given an easier item. difficulty. If you respond incorrectly, the next Most people are not skilled at estimating question you see usually will be easier than the one item difficulty, so don’t worry when taking you answered incorrectly. As you continue to the test or waste valuable time trying to determine the difficulty of the questions respond to the questions presented, the computer you are answering. will narrow your score to the number that best characterizes your ability. When you complete each section, the computer will have an accurate assessment of your ability. 8
  • 11. 1.2 What Is the GMAT®? GMAT® Test Format Because each question is presented on the basis of your answers to all previous questions, you must answer each question as it appears. You may not skip, return to, or change your responses to previous questions. Random guessing can significantly lower your scores. If you do not know the answer to a question, you should try to eliminate as many choices as possible, then select the answer you think is best. If you answer a question incorrectly by mistake—or correctly by lucky guess— your answers to subsequent questions will lead you back to questions that are at the appropriate level of difficulty for you. Each multiple-choice question used in the GMAT test has been thoroughly reviewed by professional test developers. New multiple-choice questions are tested each time the test is administered. Answers to trial questions are not counted in the scoring of your test, but the trial questions are not identified and could appear anywhere in the test. Therefore, you should try to do your best on every question. The test includes the types of questions found in this guide, but the format and presentation of the questions are different on the computer. When you take the test: • Only one question at a time is presented on the computer screen. • The answer choices for the multiple-choice questions will be preceded by circles, rather than by letters. • Different question types appear in random order in the multiple-choice sections of the test. • You must select your answer using the computer. • You must choose an answer and confirm your choice before moving on to the next question. • You may not go back to change answers to previous questions. Format of the GMAT® Questions Timing Analytical Writing Analysis of an Argument 1 30 min. Analysis of an Issue 1 30 min. Optional break 5 min. Quantitative 37 75 min. Problem Solving Data Sufficiency Optional break 5 min. Verbal 41 75 min. Reading Comprehension Critical Reasoning Sentence Correction Total Time: 210–220 min. 9
  • 12. The Official Guide for GMAT® Review 12th Edition 1.3 What Is the Content of the Test Like? It is important to recognize that the GMAT test evaluates skills and abilities developed over a relatively long period of time. Although the sections contain questions that are basically verbal and mathematical, the complete test provides one method of measuring overall ability. Keep in mind that although the questions in this guide are arranged by question type and ordered from easy to difficult, the test is organized differently. When you take the test, you may see different types of questions in any order. 1.4 Quantitative Section The GMAT Quantitative section measures your ability to reason quantitatively, solve quantitative problems, and interpret graphic data. Two types of multiple-choice questions are used in the Quantitative section: • Problem solving • Data sufficiency Problem solving and data sufficiency questions are intermingled throughout the Quantitative section. Both types of questions require basic knowledge of: • Arithmetic • Elementary algebra • Commonly known concepts of geometry To review the basic mathematical concepts that will be tested in the GMAT Quantitative questions, see the math review in chapter 4. For test-taking tips specific to the question types in the Quantitative section of the GMAT test, sample questions, and answer explanations, see chapters 5 and 6. 1.5 Verbal Section The GMAT Verbal section measures your ability to read and comprehend written material, to reason and evaluate arguments, and to correct written material to conform to standard written English. Because the Verbal section includes reading sections from several different content areas, you may be generally familiar with some of the material; however, neither the reading passages nor the questions assume detailed knowledge of the topics discussed. Three types of multiple-choice questions are used in the Verbal section: • Reading comprehension • Critical reasoning • Sentence correction These question types are intermingled throughout the Verbal section. 10
  • 13. 1.6 What Is the GMAT®? What Computer Skills Will I Need? For test-taking tips specific to each question type in the Verbal section, sample questions, and answer explanations, see chapters 7 through 9. 1.6 What Computer Skills Will I Need? You only need minimal computer skills to take the GMAT Computer-Adaptive Test (CAT). You will be required to type your essays on the computer keyboard using standard word-processing keystrokes. In the multiple-choice sections, you will select your responses using either your mouse or the keyboard. To learn more about the specific skills required to take the GMAT CAT, download the free test- preparation software available at www.mba.com. 1.7 What Are the Test Centers Like? The GMAT test is administered at a test center providing the quiet and privacy of individual computer workstations. You will have the opportunity to take two five-minute breaks—one after completing the essays and another between the Quantitative and Verbal sections. An erasable notepad will be provided for your use during the test. 1.8 How Are Scores Calculated? Your GMAT scores are determined by: • The number of questions you answer • Whether you answer correctly or incorrectly • The level of difficulty and other statistical characteristics of each question Your Verbal, Quantitative, and Total GMAT scores are determined by a complex mathematical procedure that takes into account the difficulty of the questions that were presented to you and how you answered them. When you answer the easier questions correctly, you get a chance to answer harder questions—making it possible to earn a higher score. After you have completed all the questions on the test—or when your time is up—the computer will calculate your scores. Your scores on the Verbal and Quantitative sections are combined to produce your Total score. If you have not responded to all the questions in a section (37 Quantitative questions or 41 Verbal questions), your score is adjusted, using the proportion of questions answered. Appendix A contains the 2007 percentile ranking tables that explain how your GMAT scores compare with scores of other 2007 GMAT test takers. 11
  • 14. The Official Guide for GMAT® Review 12th Edition 1.9 Analytical Writing Assessment Scores The Analytical Writing Assessment consists of two writing tasks: Analysis of an Issue and Analysis of an Argument. The responses to each of these tasks are scored on a 6-point scale, with 6 being the highest score and 1, the lowest. A score of zero (0) is given to responses that are off-topic, are in a foreign language, merely attempt to copy the topic, consist only of keystroke characters, or are blank. The readers who evaluate the responses are college and university faculty members from various subject matter areas, including management education. These readers read holistically—that is, they respond to the overall quality of your critical thinking and writing. (For details on how readers are qualified, visit www.mba.com.) In addition, responses may be scored by an automated scoring program designed to reflect the judgment of expert readers. Each response is given two independent ratings. If the ratings differ by more than a point, a third reader adjudicates. (Because of ongoing training and monitoring, discrepant ratings are rare.) Your final score is the average (rounded to the nearest half point) of the four scores independently assigned to your responses—two scores for the Analysis of an Issue and two for the Analysis of an Argument. For example, if you earned scores of 6 and 5 on the Analysis of an Issue and 4 and 4 on the Analysis of an Argument, your final score would be 5: (6 + 5 + 4 + 4) ÷ 4 = 4.75, which rounds up to 5. Your Analytical Writing Assessment scores are computed and reported separately from the multiple-choice sections of the test and have no effect on your Verbal, Quantitative, or Total scores. The schools that you have designated to receive your scores may receive your responses to the Analytical Writing Assessment with your score report. Your own copy of your score report will not include copies of your responses. 12
  • 15. 1.10 What Is the GMAT®? Test Development Process 1.10 Test Development Process The GMAT test is developed by experts who use standardized procedures to ensure high-quality, widely appropriate test material. All questions are subjected to independent reviews and are revised or discarded as necessary. Multiple-choice questions are tested during GMAT test administrations. Analytical Writing Assessment tasks are tried out on first-year business school students and then assessed for their fairness and reliability. For more information on test development, see www. mba.com. 13
  • 16. 2.0 How to Prepare 14
  • 17. 2.0 How to Prepare 2.0 How to Prepare 2.1 How Can I Best Prepare to Take the Test? We at the Graduate Management Admission Council® (GMAC®) firmly believe that the test- taking skills you can develop by using this guide—and the Verbal and Quantitative guides, if you want additional practice—are all you need to perform your best when you take the GMAT® test. By answering questions that have appeared on the GMAT test before, you will gain experience with the types of questions you may see on the test when you take it. As you practice with this guide, you will develop confidence in your ability to reason through the test questions. No additional techniques or strategies are needed to do well on the standardized test if you develop a practical familiarity with the abilities it requires. Simply by practicing and understanding the concepts that are assessed on the test, you will learn what you need to know to answer the questions correctly. 2.2 What About Practice Tests? Because a computer-adaptive test cannot be presented in paper form, we have created GMATPrep software to help you prepare for the test. The software is available for download at no charge for those who have created a user profile on www.mba.com. It is also provided on a disk, by request, to anyone who has registered for the GMAT test. The software includes two practice GMAT tests plus additional practice questions, information about the test, and tutorials to help you become familiar with how the GMAT test will appear on the computer screen at the test center. We recommend that you download the software as you start to prepare for the test. Take one practice -vs- FACT test to familiarize yourself with the test and to get an idea of how you might score. After you have – You need very advanced studied using this book, and as your test date math skills to get a high approaches, take the second practice test to GMAT score. determine whether you need to shift your focus to other areas you need to strengthen. F – The math skills tested on the GMAT test are quite basic. If you complete all the questions in this guide and The GMAT test only requires basic think you would like additional practice, you may quantitative analytic skills. You should purchase The Official Guide for GMAT® Verbal review the math skills (algebra, geometry, basic arithmetic) presented in this book, but Review or The Official Guide for GMAT® the required skill level is low. The difficulty Quantitative Review at www.mba.com. of GMAT Quantitative questions stems from the logic and analysis used to solve the problems and not the underlying math Note: There may be some overlap between this skills. book and the review sections of the GMATPrep software. 15
  • 18. The Official Guide for GMAT® Review 12th Edition 2.3 How Should I Use the Diagnostic Test? This book contains a Diagnostic Test to help you determine the types of questions that you need to practice most. You should take the Diagnostic Test around the same time that you take the first electronic sample test (using the test-preparation software). The Diagnostic Test will give you a rating—below average, average, above average, or excellent—of your skills in each type of GMAT test question. These ratings will help you identify areas to focus on as you prepare for the GMAT test. Use the results of the Diagnostic Test to help you select the right chapter of this book to start with. Next, read the introductory material carefully, and answer the sample questions in that chapter. Make sure you follow the directions for each type of question and try to work as quickly and as efficiently as possible. Then review the explanations for the correct answers, spending as much time as necessary to familiarize yourself with the range of questions or problems presented. 2.4 Where Can I Get Additional Practice? If you find you would like additional practice with the Verbal section of the test, The Official Guide for GMAT® Verbal Review is available for purchase at www.mba.com. If you want more practice with the Quantitative section, The Official Guide for GMAT® Quantitative Review is also available for purchase at www.mba.com. 2.5 General Test-Taking Suggestions Specific test-taking strategies for individual question types are presented later in this book. The following are general suggestions to help you perform your best on the test. 1. Use your time wisely. Although the GMAT test stresses accuracy more than speed, it is important to use your time wisely. On average, you will have about 1¾ minutes for each verbal question and about two minutes for each quantitative question. Once you start the test, an onscreen clock will continuously count the time you have left. You can hide this display if you want, but it is a good idea to check the clock periodically to monitor your progress. The clock will automatically alert you when five minutes remain in the allotted time for the section you are working on. 2. Answer practice questions ahead of time. After you become generally familiar with all question types, use the sample questions in this book to prepare for the actual test. It may be useful to time yourself as you answer the practice questions to get an idea of how long you will have for each question during the actual GMAT test as well as to determine whether you are answering quickly enough to complete the test in the time allotted. 3. Read all test directions carefully. The directions explain exactly what is required to answer each question type. If you read hastily, you may miss important instructions and lower your scores. To review directions during the test, click 16
  • 19. 2.5 How to Prepare General Test-Taking Suggestions on the Help icon. But be aware that the time you spend reviewing directions will count against the time allotted for that section of the test. 4. Read each question carefully and thoroughly. -vs- FACT Before you answer a multiple-choice question, – It is more important to determine exactly what is being asked, then respond correctly to the test eliminate the wrong answers and select the best questions than it is to finish choice. Never skim a question or the possible the test. answers; skimming may cause you to miss important information or nuances. F – There is a severe penalty for not completing the GMAT test. 5. Do not spend too much time on any one question. If you are stumped by a question, give it your best guess and move on. If you guess If you do not know the correct answer, or if the incorrectly, the computer program will likely give you an easier question, which you are question is too time-consuming, try to eliminate likely to answer correctly, and the computer choices you know are wrong, select the best of the will rapidly return to giving you questions matched to your ability. If you don’t finish remaining answer choices, and move on to the next the test, your score will be reduced greatly. question. Try not to worry about the impact on Failing to answer five verbal questions, for your score—guessing may lower your score, but example, could reduce your score from the 91st percentile to the 77th percentile. not finishing the section will lower your score Pacing is important. more. Bear in mind that if you do not finish a section in the allotted time, you will still receive a score. -vs- FACT 6. Confirm your answers ONLY when you are – The first 10 questions are ready to move on. critical and you should invest the most time on those. Once you have selected your answer to a multiple- choice question, you will be asked to confirm it. F – All questions count. Once you confirm your response, you cannot go back and change it. You may not skip questions, It is true that the computer-adaptive testing algorithm uses the first 10 questions to because the computer selects each question on the obtain an initial estimate of your ability; basis of your responses to preceding questions. however, that is only an initial estimate. As you continue to answer questions, the algorithm self-corrects by computing an updated estimate on the basis of all the 7. Plan your essay answers before you begin questions you have answered, and then to write. administers items that are closely matched to this new estimate of your ability. Your The best way to approach the two writing tasks final score is based on all your responses that comprise the Analytical Writing Assessment and considers the difficulty of all the questions you answered. Taking additional is to read the directions carefully, take a few time on the first 10 questions will not game minutes to think about the question, and plan a the system and can hurt your ability to finish the test. response before you begin writing. Take care to organize your ideas and develop them fully, but leave time to reread your response and make any revisions that you think would improve it. 17
  • 21. 3.0 Diagnostic Test 3.0 Diagnostic Test Like the practice sections later in the book, the Diagnostic Test uses questions from real GMAT® tests. The purpose of the Diagnostic Test is to help you determine how skilled you are in answering each of the five types of questions on the GMAT test: data sufficiency, problem solving, reading comprehension, critical reasoning, and sentence correction. Scores on the Diagnostic Test are designed to help you answer the question, “If all the questions on the GMAT test were like the questions in this section, how well would I do?” Your scores are classified as being excellent, above average, average, or below average, relative to the scores of other test takers. You can use this information to focus your test-preparation activities. Instructions 1. Take your time answering these questions. The Diagnostic Test is not timed. 2. If you are stumped by a question, you should guess and move on, just like you should do on the real GMAT test. 3. You can take one segment at a time, if you want. It is better to finish an entire section (Quantitative or Verbal) in one sitting, but this is not a requirement. 4. You can go back and change your answers in the Diagnostic Test. 5. After you take the test, check your answers using the answer key that follows the test. The number of correct answers is your raw score. 6. Convert your raw score, using the table provided. Note: The Diagnostic Test is designed to give you guidance on how to prepare for the GMAT test; however, a strong score on one type of question does not guarantee that you will perform as well on the real GMAT test. The statistical reliability of scores on the Diagnostic Test ranges from 0.75 to 0.89, and the subscale classification is about 85%–90% accurate, meaning that your scores on the Diagnostic Test are a good, but not perfect, measure of how you are likely to perform on the real test. Use the tests on the free online software to obtain a good estimate of your expected GMAT Verbal, Quantitative, and Total scores. You should not compare the number of questions you got right in each section. Instead, you should compare how your responses are rated in each section. 19
  • 22. The Official Guide for GMAT® Review 12th Edition 3.1 Quantitative Sample Questions Problem Solving Solve the problem and indicate the best of the answer choices given. Numbers: All numbers used are real numbers. Figures: All figures accompanying problem solving questions are intended to provide information useful in solving the problems. Figures are drawn as accurately as possible. Exceptions will be clearly noted. Lines shown as straight are straight, and lines that appear jagged are also straight. The positions of points, angles, regions, etc., exist in the order shown, and angle measures are greater than zero. All figures lie in a plane unless otherwise indicated. 1. Last month a certain music club offered a discount to 4. Among a group of 2,500 people, 35 percent invest in preferred customers. After the first compact disc municipal bonds, 18 percent invest in oil stocks, and purchased, preferred customers paid $3.99 for each 7 percent invest in both municipal bonds and oil additional compact disc purchased. If a preferred stocks. If 1 person is to be randomly selected from customer purchased a total of 6 compact discs and the 2,500 people, what is the probability that the paid $15.95 for the first compact disc, then the dollar person selected will be one who invests in municipal amount that the customer paid for the 6 compact bonds but NOT in oil stocks? discs is equivalent to which of the following? (A) (A) 5(4.00) + 15.90 (B) 5(4.00) + 15.95 7 (B) (C) 5(4.00) + 16.00 25 (D) 5(4.00 – 0.01) + 15.90 7 (C) (E) 5(4.00 – 0.05) + 15.95 20 21 (D) 2. The average (arithmetic mean) of the integers from 50 200 to 400, inclusive, is how much greater than the 27 (E) average of the integers from 50 to 100, inclusive? 50 (A) 150 (B) 175 5. A closed cylindrical tank contains 36π cubic feet of (C) 200 water and is filled to half its capacity. When the tank is placed upright on its circular base on level ground, (D) 225 the height of the water in the tank is 4 feet. When the (E) 300 tank is placed on its side on level ground, what is the height, in feet, of the surface of the water above the 3. The sequence a1, a2, a3,...,an,... is such that ground? an = for all n ≥ 3. If a3 = 4 and (A) 2 (B) 3 a5 = 20, what is the value of a6 ? (C) 4 (A) 12 (D) 6 (B) 16 (E) 9 (C) 20 (D) 24 (E) 28 20
  • 23. 3.1 Diagnostic Test Quantitative Sample Questions 6. A marketing firm determined that, of 200 households 1 (A) surveyed, 80 used neither Brand A nor Brand B soap, 4 60 used only Brand A soap, and for every household 1 that used both brands of soap, 3 used only Brand B (B) 2 soap. How many of the 200 households surveyed used 2 both brands of soap? (C) 3 (A) 15 (D) 2 (B) 20 (E) 4 (C) 30 (D) 40 9. A researcher computed the mean, the median, and the (E) 45 standard deviation for a set of performance scores. If 5 were to be added to each score, which of these 7. A certain club has 10 members, including Harry. One three statistics would change? of the 10 members is to be chosen at random to be the president, one of the remaining 9 members is to (A) The mean only be chosen at random to be the secretary, and one of (B) The median only the remaining 8 members is to be chosen at random (C) The standard deviation only to be the treasurer. What is the probability that Harry (D) The mean and the median will be either the member chosen to be the secretary (E) The mean and the standard deviation or the member chosen to be the treasurer? 1 (A) y° 720 1 (B) x° z° 80 1 (C) 10 1 (D) v° 9 w° 1 (E) 5 10. In the figure shown, what is the value of 8. If a certain toy store’s revenue in November was v+x+y+z+w? 2 of its revenue in December and its revenue in (A) 45 5 1 (B) 90 January was of its revenue in November, then the 4 (C) 180 store’s revenue in December was how many times the (D) 270 average (arithmetic mean) of its revenues in November and January? (E) 360 21
  • 24. The Official Guide for GMAT® Review 12th Edition 11. Of the three-digit integers greater than 700, how many 15. The product of all the prime numbers less than 20 is have two digits that are equal to each other and the closest to which of the following powers of 10 ? remaining digit different from the other two? (A) 109 (A) 90 (B) 108 (B) 82 (C) 107 (C) 80 (D) 106 (D) 45 (E) 105 (E) 36 16. If 3 − 2x = 2x + 1 , then 4x2 = 12. Positive integer y is 50 percent of 50 percent of (A) 1 positive integer x, and y percent of x equals 100. What is the value of x ? (B) 4 (C) 2 − 2x (A) 50 (D) 4x − 2 (B) 100 (E) 6x − 1 (C) 200 (D) 1,000 16 17. If n = , what is the value of ? (E) 2,000 81 1 s (A) 13. If s and t are positive integers such that = 64.12, 9 t which of the following could be the remainder when 1 (B) s is divided by t ? 4 4 (A) 2 (C) 9 (B) 4 2 (D) (C) 8 3 (D) 20 9 (E) (E) 45 2 14. Of the 84 parents who attended a meeting at a 18. If n is the product of the integers from 1 to 8, school, 35 volunteered to supervise children during inclusive, how many different prime factors greater the school picnic and 11 volunteered both to supervise than 1 does n have? children during the picnic and to bring refreshments to the picnic. If the number of parents who volunteered (A) Four to bring refreshments was 1.5 times the number of (B) Five parents who neither volunteered to supervise children (C) Six during the picnic nor volunteered to bring (D) Seven refreshments, how many of the parents volunteered to (E) Eight bring refreshments? (A) 25 19. If k is an integer and 2 < k < 7, for how many different (B) 36 values of k is there a triangle with sides of lengths 2, 7, and k ? (C) 38 (D) 42 (A) One (E) 45 (B) Two (C) Three (D) Four (E) Five 22
  • 25. 3.1 Diagnostic Test Quantitative Sample Questions 20. A right circular cone is inscribed in a hemisphere so 23. If the positive integer x is a multiple of 4 and the that the base of the cone coincides with the base of positive integer y is a multiple of 6, then xy must be a the hemisphere. What is the ratio of the height of the multiple of which of the following? cone to the radius of the hemisphere? I. 8 (A) II. 12 III. 18 (B) 1:1 1 (A) II only (C) :1 2 (B) I and II only (D) (C) I and III only (E) 2:1 (D) II and III only (E) I, II, and III 21. John deposited $10,000 to open a new savings account that earned 4 percent annual interest, 24. Aaron will jog from home at x miles per hour and then compounded quarterly. If there were no other walk back home by the same route at y miles per hour. transactions in the account, what was the amount of How many miles from home can Aaron jog so that he money in John’s account 6 months after the account spends a total of t hours jogging and walking? was opened? xt (A) (A) $10,100 y (B) $10,101 x+t (C) $10,200 (B) xy (D) $10,201 xy t (E) $10,400 (C) x+y 22. A container in the shape of a right circular cylinder x+y+t (D) 1 xy is full of water. If the volume of water in the 2 container is 36 cubic inches and the height of the y+t t (E) – container is 9 inches, what is the diameter of the base x y of the cylinder, in inches? 16 (A) 9π 4 (B) π 12 (C) π 2 (D) π 2 (E) 4 π 23
  • 26. The Official Guide for GMAT® Review 12th Edition Data Sufficiency Each data sufficiency problem consists of a question and two statements, labeled (1) and (2), which contain certain data. Using these data and your knowledge of mathematics and everyday facts (such as the number of days in July or the meaning of the word counterclockwise), decide whether the data given are sufficient for answering the question and then indicate one of the following answer choices: A Statement (1) ALONE is sufficient, but statement (2) alone is not sufficient. B Statement (2) ALONE is sufficient, but statement (1) alone is not sufficient. C BOTH statements TOGETHER are sufficient, but NEITHER statement ALONE is sufficient. D EACH statement ALONE is sufficient. E Statements (1) and (2) TOGETHER are not sufficient. Note: In data sufficiency problems that ask for the value of a quantity, the data given in the statements are sufficient only when it is possible to determine exactly one numerical value for the quantity. Example: P xº yº zº Q R In ∆PQR, what is the value of x ? (1) PQ = PR (2) y = 40 Explanation: According to statement (1) PQ = PR; therefore, ∆PQR is isosceles and y = z. Since x + y + z = 180, it follows that x + 2y = 180. Since statement (1) does not give a value for y, you cannot answer the question using statement (1) alone. According to statement (2), y = 40; therefore, x + z = 140. Since statement (2) does not give a value for z, you cannot answer the question using statement (2) alone. Using both statements together, since x + 2y = 180 and the value of y is given, you can find the value of x. Therefore, BOTH statements (1) and (2) TOGETHER are sufficient to answer the question, but NEITHER statement ALONE is sufficient. Numbers: All numbers used are real numbers. Figures: • Figures conform to the information given in the question, but will not necessarily conform to the additional information given in statements (1) and (2). • Lines shown as straight are straight, and lines that appear jagged are also straight. • The positions of points, angles, regions, etc., exist in the order shown, and angle measures are greater than zero. • All figures lie in a plane unless otherwise indicated. 24
  • 27. 3.1 Diagnostic Test Quantitative Sample Questions 25. If the units digit of integer n is greater than 2, what is 31. Is the standard deviation of the set of measurements the units digit of n ? x1, x2, x3, x4, . . ., x20 less than 3 ? (1) The units digit of n is the same as the units digit (1) The variance for the set of measurements is 4. of n2. (2) For each measurement, the difference between (2) The units digit of n is the same as the units digit the mean and that measurement is 2. of n3. 32. Is the range of the integers 6, 3, y, 4, 5, and x greater 26. What is the value of the integer p ? than 9 ? (1) Each of the integers 2, 3, and 5 is a factor of p. (1) y > 3x (2) Each of the integers 2, 5, and 7 is a factor of p. (2) y>x>3 27. If the length of Wanda’s telephone call was rounded up 33. Is ? to the nearest whole minute by her telephone company, then Wanda was charged for how many (1) 5x < 1 minutes for her telephone call? (2) x<0 (1) The total charge for Wanda’s telephone call was $6.50. 34. Of the companies surveyed about the skills they required in prospective employees, 20 percent (2) Wanda was charged $0.50 more for the first required both computer skills and writing skills. What minute of the telephone call than for each percent of the companies surveyed required neither minute after the first. computer skills nor writing skills? 28. What is the perimeter of isosceles triangle MNP ? (1) Of those companies surveyed that required computer skills, half required writing skills. (1) MN = 16 (2) 45 percent of the companies surveyed required (2) NP = 20 writing skills but not computer skills. 29. In a survey of retailers, what percent had purchased 35. What is the value of w + q ? computers for business purposes? (1) 85 percent of the retailers surveyed who owned (1) 3w = 3 − 3q their own store had purchased computers for (2) 5w + 5q = 5 business purposes. (2) 40 percent of the retailers surveyed owned their 36. If X and Y are points in a plane and X lies inside the own store. circle C with center O and radius 2, does Y lie inside circle C ? 30. The only gift certificates that a certain store sold yesterday were worth either $100 each or $10 each. If (1) The length of line segment XY is 3. the store sold a total of 20 gift certificates yesterday, (2) The length of line segment OY is 1.5. how many gift certificates worth $10 each did the store sell yesterday? 37. Is x > y ? (1) The gift certificates sold by the store yesterday (1) x=y+2 were worth a total of between $1,650 and (2) =y−1 $1,800. (2) Yesterday the store sold more than 15 gift certificates worth $100 each. 25
  • 28. The Official Guide for GMAT® Review 12th Edition 38. If Paula drove the distance from her home to her college at an average speed that was greater than 44. If m and n are positive integers, is an integer? 70 kilometers per hour, did it take her less than (1) is an integer. 3 hours to drive this distance? (1) The distance that Paula drove from her home to (2) is an integer. her college was greater than 200 kilometers. 45. Of the 66 people in a certain auditorium, at most (2) The distance that Paula drove from her home to 6 people have their birthdays in any one given month. her college was less than 205 kilometers. Does at least one person in the auditorium have a birthday in January? 39. In the xy-plane, if line k has negative slope and passes through the point (−5,r ) , is the x-intercept of line k (1) More of the people in the auditorium have their positive? birthday in February than in March. (1) The slope of line k is –5. (2) Five of the people in the auditorium have their birthday in March. (2) r>0 46. Last year the average (arithmetic mean) salary of the 40. If $5,000 invested for one year at p percent simple 10 employees of Company X was $42,800. What is the annual interest yields $500, what amount must be average salary of the same 10 employees this year? invested at k percent simple annual interest for one year to yield the same number of dollars? (1) For 8 of the 10 employees, this year’s salary is 15 percent greater than last year’s salary. (1) k = 0.8p (2) For 2 of the 10 employees, this year’s salary is (2) k=8 the same as last year’s salary. x+y 41. If > 0, is x < 0 ? z 47. In a certain classroom, there are 80 books, of which 24 are fiction and 23 are written in Spanish. How many (1) x<y of the fiction books are written in Spanish? (2) z<0 (1) Of the fiction books, there are 6 more that are 42. Does the integer k have at least three different not written in Spanish than are written in positive prime factors? Spanish. (2) Of the books written in Spanish, there are 5 k more nonfiction books than fiction books. (1) is an integer. 15 k 48. If p is the perimeter of rectangle Q, what is the value (2) is an integer. 10 of p ? 43. In City X last April, was the average (arithmetic mean) (1) Each diagonal of rectangle Q has length 10. daily high temperature greater than the median daily (2) The area of rectangle Q is 48. high temperature? (1) In City X last April, the sum of the 30 daily high temperatures was 2,160°. (2) In City X last April, 60 percent of the daily high temperatures were less than the average daily high temperature. 26
  • 29. 3.2 Diagnostic Test Verbal Sample Questions 3.2 Verbal Sample Questions Reading Comprehension Each of the reading comprehension questions is based on the content of a passage. After reading the passage, answer all questions pertaining to it on the basis of what is stated or implied in the passage. For each question, select the best answer of the choices given. Line According to economic signaling theory, 1. Which of the following best describes the purpose of consumers may perceive the frequency with the sentence in lines 10–15 ? which an unfamiliar brand is advertised as a cue that the brand is of high quality. The notion that (A) To show that economic signaling theory fails to (5) highly advertised brands are associated with explain a finding high-quality products does have some empirical (B) To introduce a distinction not accounted for by support. Marquardt and McGann found that economic signaling theory heavily advertised products did indeed rank high (C) To account for an exception to a generalization on certain measures of product quality. Because suggested by Marquardt and McGann (10) large advertising expenditures represent (D) To explain why Marquardt and McGann’s a significant investment on the part of a research was conducted manufacturer, only companies that expect to (E) To offer an explanation for an observation recoup these costs in the long run, through reported by Marquardt and McGann consumers’ repeat purchases of the product, (15) can afford to spend such amounts. However, two studies by Kirmani have found 2. The primary purpose of the passage is to that although consumers initially perceive expensive (A) present findings that contradict one explanation advertising as a signal of high brand quality, for the effects of a particular advertising at some level of spending the manufacturer’s practice (20) advertising effort may be perceived as unreasonably high, implying low manufacturer confidence in (B) argue that theoretical explanations about the product quality. If consumers perceive excessive effects of a particular advertising practice are advertising effort as a sign of a manufacturer’s of limited value without empirical evidence desperation, the result may be less favorable (C) discuss how and why particular advertising (25) brand perceptions. In addition, a third study by practices may affect consumers’ perceptions Kirmani, of print advertisements, found that the (D) contrast the research methods used in two use of color affected consumer perception of different studies of a particular advertising brand quality. Because consumers recognize that practice color advertisements are more expensive than (E) explain why a finding about consumer responses (30) black and white, the point at which repetition of an to a particular advertising practice was advertisement is perceived as excessive comes unexpected sooner for a color advertisement than for a black- and-white advertisement. 27
  • 30. The Official Guide for GMAT® Review 12th Edition 3. Kirmani’s research, as described in the passage, 5. The passage suggests that Kirmani would be most suggests which of the following regarding consumers’ likely to agree with which of the following statements expectations about the quality of advertised products? about consumers’ perceptions of the relationship between the frequency with which a product is (A) Those expectations are likely to be highest if a advertised and the product’s quality? manufacturer runs both black-and-white and color advertisements for the same product. (A) Consumers’ perceptions about the frequency (B) Those expectations can be shaped by the with which an advertisement appears are their presence of color in an advertisement as well as primary consideration when evaluating an by the frequency with which an advertisement advertisement’s claims about product quality. appears. (B) Because most consumers do not notice the (C) Those expectations are usually high for frequency of advertisement, it has little impact frequently advertised new brands but not for on most consumers’ expectations regarding frequently advertised familiar brands. product quality. (D) Those expectations are likely to be higher for (C) Consumers perceive frequency of advertisement products whose black-and-white advertisements as a signal about product quality only when the are often repeated than for those whose color advertisement is for a product that is newly on advertisements are less often repeated. the market. (E) Those expectations are less definitively shaped (D) The frequency of advertisement is not always by the manufacturer’s advertisements than by perceived by consumers to indicate that information that consumers gather from other manufacturers are highly confident about their sources. products’ quality. (E) Consumers who try a new product that has been 4. Kirmani’s third study, as described in the passage, frequently advertised are likely to perceive the suggests which of the following conclusions about a advertisement’s frequency as having been an black-and-white advertisement? accurate indicator of the product’s quality. (A) It can be repeated more frequently than a comparable color advertisement could before consumers begin to suspect low manufacturer confidence in the quality of the advertised product. (B) It will have the greatest impact on consumers’ perceptions of the quality of the advertised product if it appears during periods when a color version of the same advertisement is also being used. (C) It will attract more attention from readers of the print publication in which it appears if it is used only a few times. (D) It may be perceived by some consumers as more expensive than a comparable color advertisement. (E) It is likely to be perceived by consumers as a sign of higher manufacturer confidence in the quality of the advertised product than a comparable color advertisement would be. 28
  • 31. 3.2 Diagnostic Test Verbal Sample Questions Line The idea of the brain as an information 6. The main purpose of the passage is to processor—a machine manipulating blips of energy according to fathomable rules—has come to (A) propose an experiment dominate neuroscience. However, one enemy of (B) analyze a function (5) the brain-as-computer metaphor is John R. Searle, (C) refute an argument a philosopher who argues that since computers (D) explain a contradiction simply follow algorithms, they cannot deal with (E) simulate a process important aspects of human thought such as meaning and content. Computers are syntactic, (10) rather than semantic, creatures. People, on the 7. Which of the following is most consistent with Searle’s other hand, understand meaning because they have reasoning as presented in the passage? something Searle obscurely calls the causal powers (A) Meaning and content cannot be reduced to of the brain. algorithms. Yet how would a brain work if not by reducing (15) what it learns about the world to information—some (B) The process of digestion can be simulated kind of code that can be transmitted from neuron mechanically, but not on a computer. to neuron? What else could meaning and content (C) Simulated thoughts and real thoughts are be? If the code can be cracked, a computer should essentially similar because they are composed be able to simulate it, at least in principle. But primarily of information. (20) even if a computer could simulate the workings (D) A computer can use “causal powers” similar to of the mind, Searle would claim that the machine those of the human brain when processing would not really be thinking; it would just be acting information. as if it were. His argument proceeds thus: if a (E) Computer simulations of the world can achieve computer were used to simulate a stomach, with the complexity of the brain’s representations of (25) the stomach’s churnings faithfully reproduced on a the world. video screen, the machine would not be digesting real food. It would just be blindly manipulating the 8. The author of the passage would be most likely to symbols that generate the visual display. agree with which of the following statements about the Suppose, though, that a stomach were simulated simulation of organ functions? (30) using plastic tubes, a motor to do the churning, a supply of digestive juices, and a timing mechanism. (A) An artificial device that achieves the functions of If food went in one end of the device, what came out the stomach could be considered a valid model the other end would surely be digested food. Brains, of the stomach. unlike stomachs, are information processors, and if (B) Computer simulations of the brain are best used (35) one information processor were made to simulate to crack the brain’s codes of meaning and another information processor, it is hard to see content. how one and not the other could be said to think. Simulated thoughts and real thoughts are made of (C) Computer simulations of the brain challenge the same element: information. The representations ideas that are fundamental to psychology and (40) of the world that humans carry around in their heads neuroscience. are already simulations. To accept Searle’s argument, (D) Because the brain and the stomach both act as one would have to deny the most fundamental notion processors, they can best be simulated by in psychology and neuroscience: that brains work mechanical devices. by processing information. (E) The computer’s limitations in simulating digestion suggest equal limitations in computer- simulated thinking. 29
  • 32. The Official Guide for GMAT® Review 12th Edition 9. It can be inferred that the author of the passage believes that Searle’s argument is flawed by its failure to (A) distinguish between syntactic and semantic operations (B) explain adequately how people, unlike computers, are able to understand meaning (C) provide concrete examples illustrating its claims about thinking (D) understand how computers use algorithms to process information (E) decipher the code that is transmitted from neuron to neuron in the brain 10. From the passage, it can be inferred that the author would agree with Searle on which of the following points? (A) Computers operate by following algorithms. (B) The human brain can never fully understand its own functions. (C) The comparison of the brain to a machine is overly simplistic. (D) The most accurate models of physical processes are computer simulations. (E) Human thought and computer-simulated thought involve similar processes of representation. 11. Which of the following most accurately represents Searle’s criticism of the brain-as-computer metaphor, as that criticism is described in the passage? (A) The metaphor is not experimentally verifiable. (B) The metaphor does not take into account the unique powers of the brain. (C) The metaphor suggests that a brain’s functions can be simulated as easily as those of a stomach. (D) The metaphor suggests that a computer can simulate the workings of the mind by using the codes of neural transmission. (E) The metaphor is unhelpful because both the brain and the computer process information. 30
  • 33. 3.2 Diagnostic Test Verbal Sample Questions Line Women’s grassroots activism and their vision 12. The primary purpose of the passage is to of a new civic consciousness lay at the heart of social reform in the United States throughout the (A) explain why women reformers of the Progressive Progressive Era, the period between the depression Era failed to achieve their goals (5) of 1893 and America’s entry into the Second (B) discuss the origins of child labor laws in the late World War. Though largely disenfranchised except nineteenth and early twentieth centuries for school elections, white middle-class women (C) compare the living conditions of working-class reformers won a variety of victories, notably in and middle-class women in the Progressive Era the improvement of working conditions, especially (D) discuss an oversight on the part of women (10) for women and children. Ironically, though, reformers of the Progressive Era child labor legislation pitted women of different (E) revise a traditional view of the role played by classes against one another. To the reformers, women reformers in enacting Progressive Era child labor and industrial home work were equally reforms inhumane practices that should be outlawed, but, (15) as a number of women historians have recently observed, working-class mothers did not always 13. The view mentioned in line 17 of the passage refers to share this view. Given the precarious finances of which of the following? working-class families and the necessity of pooling (A) Some working-class mothers’ resistance to the the wages of as many family members as possible, enforcement of child labor laws (20) working-class families viewed the passage and enforcement of stringent child labor statutes as a (B) Reformers’ belief that child labor and industrial personal economic disaster and made strenuous home work should be abolished efforts to circumvent child labor laws. Yet (C) Reformers’ opinions about how working-class reformers rarely understood this resistance in terms families raised their children (25) of the desperate economic situation of working- (D) Certain women historians’ observation that there class families, interpreting it instead as evidence was a lack of consensus between women of of poor parenting. This is not to dispute women different classes on the issue of child labor and reformers’ perception of child labor as a terribly industrial home work exploitative practice, but their understanding of (E) Working-class families’ fears about the adverse (30) child labor and their legislative solutions for ending consequences that child labor laws would have it failed to take account of the economic needs of on their ability to earn an adequate living working-class families. 14. The author of the passage mentions the observations of women historians (lines 15–17) most probably in order to (A) provide support for an assertion made in the preceding sentence (lines 10–12) (B) raise a question that is answered in the last sentence of the passage (lines 27–32) (C) introduce an opinion that challenges a statement made in the first sentence of the passage (D) offer an alternative view to the one attributed in the passage to working-class mothers (E) point out a contradiction inherent in the traditional view of child labor reform as it is presented in the passage 31
  • 34. The Official Guide for GMAT® Review 12th Edition 15. The passage suggests that which of the following was 17. According to the passage, one of the most striking a reason for the difference of opinion between achievements of white middle-class women reformers working-class mothers and women reformers on the during the Progressive Era was issue of child labor? (A) gaining the right to vote in school elections (A) Reformers’ belief that industrial home work was (B) mobilizing working-class women in the fight preferable to child labor outside the home against child labor (B) Reformers’ belief that child labor laws should pertain to working conditions but not to pay (C) uniting women of different classes in grassroots activism (C) Working-class mothers’ resentment at reformers’ attempts to interfere with their parenting (D) improving the economic conditions of working- (D) Working-class mothers’ belief that child labor class families was an inhumane practice (E) improving women’s and children’s working (E) Working-class families’ need for every conditions employable member of their families to earn money 16. The author of the passage asserts which of the following about women reformers who tried to abolish child labor? (A) They alienated working-class mothers by attempting to enlist them in agitating for progressive causes. (B) They underestimated the prevalence of child labor among the working classes. (C) They were correct in their conviction that child labor was deplorable but shortsighted about the impact of child labor legislation on working-class families. (D) They were aggressive in their attempts to enforce child labor legislation, but were unable to prevent working-class families from circumventing them. (E) They were prevented by their nearly total disenfranchisement from making significant progress in child labor reform. 32